Showing posts with label FCPS 1 OB/GYN MCQ POOL. Show all posts
Showing posts with label FCPS 1 OB/GYN MCQ POOL. Show all posts

11.8.15

FCPS Gynae Obs Paper part 1 June 2014

FCPS Gynae Obs Paper part 1 June 2014


Kindly correct the answers if found wrong, thanks

1) ureter is narrow at the following points except?
pelvic brim
pelvic ureteric junction where it lies on the psoas (ans)
where it passes through the bladder

2)regarding spinal nerves true statemnt?
they leave the vertebral canal via intervertebral foramina(ANS)
dorsal root contain both sensory & motor fibers
have ganglia on there ventral root

3) following statements about ischio rectal fossa are true except?
has pudendal canal in lower medial wall (ANS)
allows dilatation of anal canal during defecation
extend forward into urogenital triangle
bounded lat by obturatr internus
is bounded post by sacrotuberous ligamnt

4) regarding suprarenal gland all true excpt?
foetal gland is 1/3rd the size of kidney(ANS)
cortex develop frm mesoderm

5) following structures present in dermis except?
melanocytes(ANS)
fibroblast
macrophages
reticulocytes
mast cells

6) neural crest cell give

11.11.11

FCPS PAST PAPERS Part -1 (Medicine, Surgery, Radiology, OB/GYN) subject-wise pool with answers

01. A 10 year old boy with shoulder injury lost abduction of his shoulder up to 30 degree which muscle is lost:
Supraspinatus
Deltoid?

02. A boy is brought to a hospital with injury at elbow joint lost extension of medial 4 fingers at MTP joint, abduction of thumb with intact sensation due to:
Injury to Radial nerve at elbow

03. Which of the following muscle is not supplied by median nerve:
Adductor polices.

04. Regarding palm:
Superficial palmer arch is below palmer apeneurosis.

05. Which of the following muscle have dual nerve supply from median and ulner nerve:
Flexor digitorum profundus.

06. Infection of the first pulp space lymph nodes will first drained.
Epitrochlear lymph nodes.

07. Which of the following muscles is flexor at hip and extensor at knee:
Sartorius.

08. In an injury to knee joint a man is unable to extend his knee the root value affected:
L3-L4.

09. In the injury to neck of fibula the artery damaged:
Ant. Tibial artery.
Peroneal artery. (correct)

10. Regarding post. Triangle:
Base is form by middle 3rd of clavicle.

11. Bifurcation of common carotid artery:
Beneath the ant. Border of sternocladomastoid at the sup. Border of thyroid cartilage.

12. Great saphenous vein has how many valves:
20.

13. Vertebral venous plexus:
Has no communication with cerebral sinuses.
Lies in epidural space.
Is formed by two vertebral veins.

14. A 40 years old man feels pain in his gastrocnemeus muscle after he walks 100 meter which relieves when he stops walking is likely due to obstruction of:
A. Post. Tibial artery.

15. Subclavian artery grooves over:
A. 1st rib.

16. What is inappropriate about azygus vein.
A. It enters the thoracic cavity thru esophageal opening.

17. Which of the following structure arches over root of left lung:
A. Arch of aorta.

18. Weakest point of rib is:
A. Angle of the rib.

19. A pathologist wants to pass dye to the liver he will reach thru:
A. Ligamentum Teres.

20. Internal spermatic fascia is a continuation of:
A. Transversalis fascia.

21. The branch of internal carotid artery which remains in true pelvis throughout its course:
A. Middle rectal artery.

22. During a per rectal examination the examiner will not reveal:
A. Ureter

23. Parasympathetic nerves supplying the urinary bladder are:
A. Pelvic splanchnic nerves.

24. Nerve supply to the lower airways:
A. T3-4

25. During an injury to the bulbar part of urethra the urine will extravasate to:
A. Superficial perineal pouch.

26. Nerve supply to the distal scrotum is thru:
Illioinguinal nerve. (correct)
Illiohypogastric nerve.
Genitofemoral nerve.

27. During surgery of right colon for carcinoma which structure would not be injured:
A. Aorta.

28. Slow growing tumor of head of pancreas will compress:
A. Common Bile Duct.

29. Tumor of the head of pancreas will compress:
A. Common Bile Duct.

EMBYROLOGY.

1. Regarding Decidua:
A. Decidua basilis forms the maternal part of placenta.

2. Embryonic period is:
A. 3-8 weeks.

3. Which of the following is a derivative of neuro ectoderm:
A. Sphincter pupili muscle.

4. Derivative of Ectoderm include:
A. Secretary Epithelium of parotid gland.

6. Regarding Allantois:
……

7. left umbilical vein will regress to:
A. Ligamentum Teres.

8. Which of the following structure is not an Embryological remnant:
A. Lateral umbilical ligament.

9. Regarding Mesothelium:
A. Lines the body cavities.

10. Physiological hernia occurs in between:
A. 6-10 weeks.

11. What will happen immediately after birth:
Anatomical closure of Ductus arteriosus.
Anatomical closure of Ductus venosus.
Anatomical closure of foramen ovale. (correct)
Obliteration of Left umbilical vein.
Obliteration of Right umbilical vein.

12. Which of the following bone is derived from 2nd Pharyngeal arch:
A. Styloid process.

13. Muscles innervated by hypoglossal nerves are derived from:
A. Occipital Somites.

14. Primordial Germ cells are derived from:
A. Yolk sac endoderm.

PHYSIOLOGY

1. Pulmonary Wedge Pressure:
A. Indirectly measures left atrial pressure,
2. Ventricular Depolarization on ECG strip is represented by:
A. QRS Complex.
3. Ventricular preload is measured by:
A. LVEDV.
4. Blood flow to the left ventricles is increase by:
Acetylcholine infusion.
Sympathetic stimulation (correct)
5. Bain Bridge reflux.
……………
6. IPSP is generated by opening of:
A. Cl- channels.
7. During Depolarization:
A. There is rapid influx of Na+.
8. Blood group antigen:
A. Will not found in any other tissue than blood.
B. Will be inherited by Autosomal recessive pattern
9. If father’s blood group is B+ve & mother’s blood group is AB+ve there child can not have which of the following blood group:
A. O+ve.
10. ESR will increase with the decrease in:
A. Albumin.
11. ADH will act on:
A. Distal Tubules.
12. Renal absorption of glucose thru secondary transport with sodium occurs at:
A. Proximal Tubules.
12. Simple Diffusion depends on all of the following factors except:
A. Magnetic field.
13. What is the suitable I/V fluid for the patient of acidurea:
Normal Saline. (correct)
Dorrow`s solution.
Ringer lactate.
5% dextrose
10% dextrose.
14. Which of the following causes Hyperkelemia:
Zollinger Ellison syndrome.
Cushing syndrome.
Conn`s syndrome. ????
Pyloric stenosis. ?????
15. Which of the following hormone is called STRESS HORMONE:
A. Cortisol.
16. Inhibitory factor released by hypothalamus against which of the following hormone:
Prolactin
Growth hormone.
17. Bile of the Liver differs from the GB bile because GB bile contains decrease amount of:
A. Water.
18. Which of the following cell can never reproduce:
Erythrocyte.
Neuron.
Skeletal muscle.
Smooth muscle. (correct)
19. Which of the following is the NOT non dividing cell:
A. Hepatocytes.
20. Heat loss mainly depends on:
Temperature of the surrounding. (correct)
Evaporation.
21. Cortisol decreases which of the following cell:
A. Lymphocyte.
22. Resection of distal ileum will impair the absorption of:
A. Bile Salts.
23. Regarding Bradykinin:
It formation is activated by killkeran.
24. Regarding Interferon:
……….
25. Surfactant is released by:
A. Type II pneumocytes.
26. Increase peripheral resistance is due to:
A. Increase vasomotor tone.
27. Chloride shift means:
A. Transfer of Cl- in erythrocytes in exchange of HCO3.
28. Cardiac muscles are prevented by tetanization due to its:
Rythmicity
Automaticity
Conductivity
Long refractory period. (correct)
29. Sustained rhythmic reflex tremors induce by sudden movements is:
A. Clonus.
30. Most active form of thyroid hormone present in circulation is:
A. T3.
31. Regarding conduction of visual pathways:
………..
32. Regarding olfaction:
A. Sharp odors have the quality of water & lipid solubility.
33. Unilateral Anosmia is due to:
A. Affection of Nasal mucosa.

GENERAL PATHOLOGY

1. Irreversible cell injury starts with:
A. Rupture of the Lysosomal membrane.
2. Which of the following is not a mediator of acute inflammation:
A. Dopamin.
3. ICAM & VCAM mediates:
A. Leukocyte Adhesion.
4. Fever in inflammation is caused by:
A. IL1 & TNF α.
5. A 36 years old women presented with acute abdomen. At laparoscopy most of the bowl loops were dark purple black. Her mesenteric veins were patent. The most probable underlying pathological process is:
A. Wet Gangrene.
6. Superimposed infection on necrosis is called:
A. Gangrenous Necrosis.
7. Fluid accumulates in acute inflammation contains:
A. Proteins >3gm/dl.
8. Fat necrosis occurs in:
A. Acute pancreatitis.
9. What is inappropriate about fat necrosis:
A. Only caused by trauma to the fat tissue.
10. A 25 year old lady with 14 weeks pregnancy had a road traffic accident brought to the emergency department with a large open wound on thigh and femur fracture. After 2 days of accident she suddenly collapsed & died. The most probable pathological process involve in her death:
A. Fat Embolism.
11. Regarding Fat Embolism:
It is Fetal in >80% of cases.
IT manifest within 12 hours.
12. A 30 year old women brought to emergency department with bleeding…..


13. A 16 years old boy deeply jaundiced presented with gum bleeding due to:
A. Vitamin K deficiency.
14. A patient presented with enlarge lymph nodes, the diagnosis of Tuberculosis in this patient is confirmed by:
A. Presence of caseous necrosis in lymph nodes.
15. An 18 year old girl came to family doctor complaining of primary amenorrhea, on examination she found to have well formed breasts, blind ended vagina with no uterus & ovary. Her probable Karyotype is:
A. 46 XY.
16. Edema due to increase hydrostatic pressure is seen in:
A. Congestive cardiac failure.
17. Which of the following is a benign tumor:
A. Warthin`s tumor.
18. Which of the following is a premalignant condition:
A. Leukoplakia.
B. Erythroplakia.
19. Which of the following is a premalignant condition:
Condyloma
Metaplasia Endocervix.
Cystic hyperplastic endometrium
20. Metaplasia does not occur in:
Brain.
Vitamin A deficiency.
21. The primary source of Creatinin is:
Liver.
RBC.
Skeletal muscles.
Cardiac muscles.
Lungs.
22. Dystrophic Calcification is seen in all of the following except:
Malarial parasite.
Dead fetus.
Hydatid Cyst.
23. Maximal tensile Strength of a wound is attained in.
3 Months.
Over an unpredictable period of time.
1 Year.
6 days.
10 days.
24. Regarding autosomal recessive disorders:
………………
25. Which of the following are not non-dividing cells.
A. Hepatocytes.
26. In a patient there is atrophy of submandibular gland due to its duct obstruction. Atrophy of the gland is because of:
A. Apoptosis.
27. The mechanism of injury of ionization radiation is:
A. Free radical formation.
28. LASER act by:
Cutting.
Biochemical changes
29. A patient admitted to the hosp. diagnosed as case of gas gangrene dies due to:
A. Toxic shock
30. Which of the following is the confirmatory test of AIDS:
Western blot.
ELISA.

SPECIAL PATHOLOGY
1. Hepatic hemengioma is associated with:
A. Vinyl Chloride.
2. Which of the disease is common in I/V drug abusers:
A. Infective Endocarditis.
3. Typhoid carriers are:
A. Usually asymptomatic.
4. Intestinal metaplasia due to reflux esophagitis leads to:
A. Adenocarcinoma.
5. Embryological marker that reappears in circulation in Ca Colon is:
A. CEA.
6. Which of the following is peculiar for crohn`s disease is:
A. Perianal Lesions.
7. Anemia of pregnancy is due to:
A. Increase plasma volume.
8. Major complication of severe burn:
Formation of granulation tissue.
Carcinoma formation.
9. What is Inappropriate about hyper parathyroidism:
A. There is increase PO4 renal absorption.
10. A 25 year old lady complaining of palpitations, heat intolerance on examination her pulse is 112b/min, BP 120/80, R/R 20 br/min. She is most probably a case of:
A. Hyperthyroidism.
11. Resection of Anterior lobe of pituitary will lead to:
A. Decrease glucocorticoids.
12. A 6 year old boy is complaining of proximal muscular weakness was found to be Ca++ channel antibodies positive. The most probable diagnosis is:
Lambert Eaten Syndrome.
Mysthenia Gravis.
13. In pulmonary embolism, respiratory failure is due to:
A. Ventilation / perfusion mismatch.
14. Left ventricular failure will lead to:
A. Increase pulmonary arteriolar Pressure.
15. Immediate effect after injury to a vessel is:
Vasoconstriction
16. Effects of hyper parathyroidism in Ca. lung is due to:
Parathyroid like protein
Parathyroid hormone
17. Spleenectomy will help in treating which type of anemia:
A. Hereditary Spherocytosis.
18. Malignant Tumor that will not metastasize:
A. Basal cell carcinoma.
19. Achlasia is due to:
A. Absence of Myenteric plexus.
20. After a traffic accident a lady is brought to a hosp the IMMEDIATE step you do:
A. Clear airway.
21. Ameobic infection reaches lungs via:
Direct extension from liver.
By aspiration of ova of Entameoba histolitica.
By aspiration of trophozoits of Entameoba histolitica.
Via Portal vein.
22. In Hypothyroidism there is increase in:
A. Cholesterol
23. In thyroid disease the antibodies are directed against:
A. Thyroglobin.
24. A 50 years old man is awakened in the middle of the night with acute pain & tenderness of the right knee. He has a low grade fever. His knee is hot, tender & swollen. Analysis of fluid from right knee shows: leukocytes 70,000/mm3 with 75% neutrophils, crystal analysis = negative birefringent, gram stain = negative. The most likely diagnosis is:
A. Gout.
B. Pseudo gout.
C. Septic arthritis. (correct)

PHARMACOLOGY
1. Which of the following blocks α and β receptors:
A. Labetalol.
2. Ketamin is used as anesthetic in repeated dressings of burn patient because:
A. It relieves pain as well.
3. Drug that decreases the tone of lower esophageal sphincter & increases gastric emptying:
A. Metaclopromide.
4. Which of the following is suitable antihypertensive for asthma & IHD patient during surgery:
I/V sodium nitroprusside.
I/V nitroglycerin.
5. Which of the following is drug of choice for patient of status asthematicus:
I/V aminophylin
Oral steroid
I/V salbutamol.
6. During surgery antibiotics should be given at:
A. At the time of induction of anesthesia.
7. Heparin will inhibit:
A. Clot propagation.
B. Clot organization.
8. Acid suppression is done by blocking:
A. H2 Receptor.
9. Drug of choice for acute pancreatitis.
Pethidine.
Morphine.
Paracetamol
9] A male with Gynaecomastia, small testes, diagnosed as Klienfelters syndrome. The karyotype involved is

A} 45 XO
B} 46 XX
C} 47 XXX
D} 47 XXy (correct)
E} 47 XX

10] Somites are derived from

A} Intermediate mesoderm
B} Paraxial mesoderm
C} Lateral plate mesoderm (correct)
D} Ectoderm
E} Endoderm

11] An I/V Urograph of 8 year old boy shows Excretion of dye normal I kidney but absence of shadow on left side. Instead there is a small shadow just above the bladder on the left side. Which one of the following is most likely?

A} Horse shoe kidney
B} Constricted ectopic kidney
C} Pelvis kidney
D} Non rotating kidney
E} Unilateral renal agenesis

12] Which one of the following is not formed from urogenital sinus?

A} Bartholin’s gland
B} Urethral glands
C} Para urethral glands
D} Seminal vesicle
E}

13] Virus damage the cell by

A} making the pores in cell membrane
B} Forming free radicals
C} Altering the formation of proteins c
D} Nuclear damage
E} using energy for their own metabolism

14] The tissue damage by Ionization Radiation is due to
A) Damage to Golgi bodies
B} Formation of free radicals
C} Hydropic degeneration
D} Metaphase of cell
E} Swelling of cells
15] All are features of IRREVERSIBLE cell injury EXCEPT
A} Karyolysis
B} Karyorhexis
C} Autolysis
D} Shrinkage of mitochondria
E} Appearance of myelin figure
16] Which one of the following is least likely involved in increasing Osteoporosis

A} Increased wt: on bone
B} Renal cell carcinoma
C} Carcinoma prostate
D}
E}

17] Patient X-Ray shows hilar lymph nodes. Granuloma but no necrosis. What is the diagnosis?

A} Sarcoidosis (correct)
B} Silicosis
C} Tuberculosis
D}
E}

18] In women the most common form of necrosis after trauma is due to

A} Trauma to fatty tissue
B}
C}
D}
E}

19] Dystrophic calcification is present in which of the following

A} Acute pancreatitis
B} Malaria
C} Tuberculosis (correct)
D}
E}

20] Which one of the following is most likely feature of reversible injury?

A} Cell swelling
B} Karyorhexis
C} Decreased glycogen
D} Myelin figures (correct)
E}

21] A person fell on road and got an abrasion on elbow the first event likely occur would be

A} Vasoconstriction (correct)
B} Platelet adhesion
C} Platelet aggregation
D}
E}

22] Which one of the following facilitates phagocytosis in neutrophils?

A} C5a
B} C3a
C} C3b (correct)
D} C3 - C5 complex
E}

23] Which one of the following is potent antioxidant?

A} Vitamin A
B} Vitamin C
C} Vitamin E (correct)
D} Transferrin
E}

24] Which one of the following among Arachidonic acid metabolite is most likely function mediated in phagocytosis?

A} Chemo taxis (correct)
B} Diapedisis
C} Transmigration
D}
E}

25] Which one of the following is potent COX-II INHIBITOR?

A} Aspirin
B} Celocoxib (correct)
C} Indomethacin
D} Meloxicam
E} Piroxicam
26] In case of pulmonary embolism extensive infarction causes sudden death due to

A} Air embolism
B} Fat embolism
C} Thromboembolism (correct)
D}
E}

27] A patient was operated for abdominal surgery. A few days after he developed pain and swelling in his right leg. Which one of the following is most likely?

A} endothelial injury
B} hypercoagubility
C} stasis and endothelial injury (correct)
D}
E}

28] A soldier has profuse bleeding. He was admitted in the hospital. Which one of the following should be given immediately?

Fresh frozen plasma
B} 20 days old blood
C} 3 days old blood (correct)
D}
E}

29] A boy was born with the condition of polycystic kidney disease. Which one of the following is correct?

A} Autosomal Dominant disorder
B} Autosomal recessive disorder (correct)
C} X- linked disorder
D}
E}

30] In a renal transplantation, which one of the following is single most significant test to be done?

A} ABO blood grouping of donor and recipient
B} HLA typing and matching (correct)
C} Donor and recipient should be close relatives
D}
E}

31] A organ was transplanted to the patient, but within 10 mins the organ failed. Which one of the following is likely etiology?

A} Antibody mediated
B} T-Cell mediated
C} Preformed antibodies (correct)
D}
E}

32] AIDS occur in the presence of

A} Kaposi’s sarcoma
B}
C}
D}
E}

33] A 2 year old boy is suspected as HIV positive, which one of the following is the confirmatory test in the boy

A} ELISA test
B} Western Blot test (correct)
C} PCR
D} Increased levels of HIV in T lymphocytes
E}

34] A 10 year old boy was operated for gastrectomy, which one of the following type of anemia is developed postoperatively

A} Megaloblastic anemia (correct)
B} Iron deficiency anemia
C} Normocytic norm chromic anemia
D}
E}

35] Autoimmunity is not involved in which one of the following

A} Pernicious anemia
B} Grave’s disease
C} Diabetes mellitus
D} Hashimoto’s thyroiditis
E} Thymoma (correct)

36] In case of hashimoto’s thyroiditis, which one of the following is seen?

A} Anti thyroglobulin
B}
C}
D}
E}

37] To differentiate Chron’s disease from Ulcerative colitis, which one of the following is likely significant regarding Chron’s disease

A} has increased frequency to cancer
B} Involves both small and large intestine (correct)
C}
D}
E}

38] Regarding alcoholic hepatitis, following is finding

A} Mallory bodies
B}
C}
D}
E}

39] A 20 year old girl has developed Galactorrhea due to prolactinoma. Bromocriptine is given to treat it the drug causes

A} Inhibition of prolaction from pituitary gland (correct)
B} Decreased prolaction from hypothalamus
C} Inhibition of prolaction from breast
D} Stimulation of prolaction of anterior pituitary
E}

40] An unconscious patient presented in ED does not respond to Naloxone. Which of the following drug has been taken by this patient?

A} Morphine
B} Heroine (correct)
C} Pethidine
D} Phenobarbital
E}

41] Strongest layer of small intestine is

A} Mucosa
B} Sub mucosa
C} Circular layer
D} Serosa
E} Longitudinal layer

42] Regarding adrenal medulla which one of the following is true?

A} Secretes epinephrine and Norepinepinephrine in 80/20 ratio
B}
C}
D}
E}

43] R.E.R most likely

A} Synthesizes protein (correct)
B} Synthesizes lipids
C}
D}
E}

44] Half life of insulin is

A} 5 mins
B} 90 mins
C} 120 mins
D}
E}

45] Insulin

A} Converts glycogen to glucose
B} Stimulates protein synthesis (correct)
C} Stimulates lipolysis
D}
E}

46] Which one of the following most likely contains abundant cholesterol?

A} VLDL
B} LDL (correct)
C} Chylmicrons
D} HDL
E}


47] Which one of the following most likely causes Increase in GFR?

A} Constriction of afferent arteriole
B} Constriction Of efferent arteriole (correct)
C}
D}
E}

48] Gastric emptying is delayed by

A} Gastrin
B} Secretin
C} CCK (correct)
D}
E}

49] Which one of the following pancreatic enzyme is most likely involved in fat absorption

A} Amylase
B} Lipase
C}
D}
E}

50] Smooth muscle

A} Contain sarcomere
B} T-tubules are present
C} Gap junctions are present (correct)
D} Forms motor end plate
E}

51] Steady pressure on skin is perceived by

A} Pacinian’s corpuscle (correct)
B} Ruffini’s corpuscle
C} Meissener’s Corpuscle
D} Golgi tendon organs
E}


39. Metabolic alkalosis a. renal failure

40. ARF- most effect.a. PCT a b. DCT c. LOH d. CD

41. Hb 8.0, HCO3- inc (35), pH Inc, Ca++ dec. Cause a. ARF b. Vit D def c. folic acid deficiency d. PTH

43. High output cardiac failure
a. Thiamine a b. B2 c. B6 d. Folic acid

44. 90 yrs old lady with purplish large patch on Rt hand and arm. No itching or pain. No comorbids, Otherwise Normal. Reason? a. Capillary abn
b. Prothrombin c. Vit K dependent clotting factors

47. Warfarin, effects on: a. Factor VIII b. factor III c. Prothrombin c

48. Ibpratopium bromide given by nebulization: a. dec ACh release in bronchi a b. vasodil c. inc secretion

49. Mast cell stabilizer. a. Na chromoglycate
52. Sec released from thyroid gland entering circulation is: a. Thyroxine b. Free T3 c. free T4 d. TSH e. TRH

53. Cushing's triad: a. Inc ICP, HTN, Bradycardia b. Inc ICP, hypoTN, bradycardia c. inc ICP, hypoTN, tachycardia d. inc ICP, HTN. tachycardia e. dec ICP, HTN, tachycardia

122. Pt complains of chest pain for more than 30 min . ECG shows changes in V1- V4. It denotes: a. anterior wall MI a b. anterolateral MI c. Inferior wall MI d. Lateral wall MI e. Inferior wall MI

150. Medullary respiratory group of neurons t of ascending reticular system a. True b. false 1

151. Anion gap increases in: a. Uncontrolled DM

152. Bile from liver is different from that from GB in:

160. Mediators of inflammation a. C3a b. C5a b c. IL1 d. TNF alpha
161. Largest total cross-sectional and surface area is of: a. artery b. arterioles c. capillaries d. venules e. veins e

162. Blood flow regulation is mediated by: a. arterioles a b. precepillary sphincter c. capillaries d. venules
4] A 16 year old boy was given CO-TRIMAXAZOLE for fever, after few hours he developed red color urine, likely abnormality is in A} Glucose}-6-Phosphate deficiencyB} Immune hemolysis C} oxysmal Nocturnal Haemoglobinuria D} Malaria d
E} Sickle cell disease

5] Thiamine deficiency causes A} Peripheral Neuropathy a B. PellagraC} Chelosis D} Dermatitis

6] Milk is notoriously deficient in A} Vitamin C B} Iron C
Riboflavin D} Pantothenic acid E} Vitamin A

7] Dry heat kills the bacteria by A} Coagulation of proteins B} Cell lysis C} Free radical formation D} Direct killing d E} Oxidation

8] Virus damage the cell by A} Making the pores in cell membrane B} Forming free radicals C} Altering the formation of proteins c D} Nuclear damage E} using energy for their own metabolism

9] Which one of the following lipoprotein has highest protein content? A} HDL} a} B} LDL C} VL} D} IDL E} CHYLMICRONS

10] All are related to PLATELET FUNCTION EXCEPT A}
Capillary resilience test a B} BT C} Clot retraction

11] ALLELLE is: A} Fragmented genes B} Non-identical genes C} Structural genes D} Regulatory gene E} Autosomal Dominant gene

12] D.I.C is stimulated by: A} Factor X B} Factor XII C} Thromboplastin D} Factor IX d E} Fibrinogen

13] In enchymal cells Iron is deposited in the form of A} Tranferrin B}
Ferritin bC} Heamatin D} Heamoglobin E} Lipifuscin

19] Regarding Von Willbrand disease A} Autosomal Dominant B} Autosomal Recessive C}
Some family members have low level of Factor IX D} Some family members have abnormal level of Factor IX E} has positive family history

20] Which of the following condition is related to lymphatics? A} Angiosarcoma a B} Rhabdomyosarcoma C} Cystic hygroma

21] The condition in which pulmonary trunk fails to divide leading A} PDA aB} Truncus arteriosus C} TOF

22] Basal Cell Carcinoma is located at A} Lower lip B} Upper lip C} Tongue D} Hard palate E} Soft palate

25] Which of the following is not Carcinogen? A} Alcohol B} Benzathracine C}
Cyclophosphamide D} Benzidine E} Dimethylsulfate

26] The Initial step in APOPTOSIS is A} Inhibition of P53 genes B} Activation of BCL-2 C} Activation of Caspases c D} pores formed by lymphocytes

28] Which t of kidney produces ERYTROPOITEN A} Macula Densa B} Peritubular Capillary b C} PCT D} DCT E} LOH

29] A patient with signs and symptoms of megaloblasstic anemia. The likely cause is
A Decreased GIT motility B} Deficiency of intrinsic factor b C} Malabsorption

30] Stimulation to increase Serum Calcium is A} Hypo-phosphatemia B} atharmone b C} Decreased ECF

31] Malnutrition causes all of the following EXCEPT A} Pitting Edema. a B} Loss of subcutaneous fat. C} Failure to thrive. D} Skin pigmentation E} Thrombocytopenia
32] Liver biopsy from a pati
ent shows a lesion which comprises of central collection of structure less material surrounded by epitheliod cells and lymphocytes. One large cell with multiple nuclei arranged close to membrane is also seen. The most likely change in central area would b
e: A} Caseous necrosis B} Coagulation necrosis C} Fatty changesD} Hydropic changes E} Liquefective necrosis

33] A 28 year fisherman presented with slowly developing lethargy, easy fatigue and palpitations. His diet comprises of fish an
d rice only. Physical examination: Pallor, Loss of touch sense in both feet and legs Hb: 7.5 gm/dl, Platelet count: 110 x 10 9/L. Reticulocyte count: 0.5%. Stool DR: Ova of Intestinal asite. The most likely causative asite is A}
Ankylostomata Duodenale B} Ascris Lumbricoidis C} Diphylobothrium Latum c D} Strangyloids Stercoralis E} Taenia Saginata

34] A 40 year old male after road traffic accident had received massive blood transfusion. He is expected to have all EXCEPT A} Hypocalcaemia B} Hypokalemia b C} Hypothermia D}
Left shift of Oxyheamoglobin dissociation curve E} Metabolic acidosis

35] The endothelial molecules ICAM-1 & VCAM-1 are responsible for A} Direct endothelial injury B} Endothelial cell contraction C} Junctional re
Traction D} Leukocyte adhesion d E} Leukocyte margination

36] Eating raw or under cooked meat can cause infestation by A} Dracunculous Medinensis B} Echinecoccus Granulosis C Liver Fluke D
Schistosomiasis E} Tape Worm e

37] A pt: has sustained burn that is very painful with blisters. Healing of this burn will take place from which of the following layers of the skin A} Basal layer of epidermis}
Keratinized layer C} Layer of upper dermis c D} Lower dermis E} Subcutaneous layer

38]
A 40 year old male presented with jaundice and generalized itching. His LFT shows Direct Billirubin 11 mg/100 ml, SGPT 75U/L, Alka Phosphate 2300 IU/L 500IU/L. The most likely cause is A} Drug induced jaundice B}
Extra hepatic Cholestasis b C} Autoimmune Hepatitis D} Alcoholic Hepatitis E} Intrahepatic Cholestasis

39] The tissue damage by Ionization Radiation is due to A} Damage to Golgi bodies B}
Formation of free radicals b C} Hydropic degeneration D} Metaplasia of cell E} Swelling of cells

42] Increase in Light Chain Immunoglobulins Is the characteristic feature of
A} Endocrine type Amyloidosis B} Hereditary Amyloidosis C} Localized type Amyloidosis D} Reactive Amyloidosis E} Systemic type Amyloidosis

43] Chemotaxis is caused by A} C5a a B} C5b C} Lymphokines D} Leucotriene B3 E} Opsonins

44] Following are the actions of Archidonic acid metabolites in inflammation EXCEPT A} Chemotaxis
B} Increased vascular perm

FCPS PAST PAPERS POOL PART-1 OB/GYNA with answers


question no 46:mothers milk differs from cows milk in that it contains:
a)calcium
b)lactose (correct)
c)proteins
d)vitamin D

question no 48:patient has endocrine disorder has metabolic alkalosis and hypokalemia,he has excess of:
a)ACTH
b)aldosterone (correct)
c)cortisol
d)estrogen
e)progesterone

question no 49:hormone that stores carbohydrates, lipids n proteins in cells is also known as hormone of abundance:
a)growth hormone (correct)
b)cortisol
c)thyroid hormone
d)estrogen
e)progesterone

question no 50:hyperglycemia is caused by:
a)insulin
b)glucagon (correct)
c)estrogen
d)progesterone
e)growth hormone

question no 51:ketoacidosis is caused by:
a)increased production of glucose
b)decreased production of glucose (correct)
c)decreased utilisation of carbohydrate in body
d)insulin excess

question no 52:insulin secretion is stopped by
:a)glucagon
b)decreased k levels
c)somatostatin (correct)

question no 53:hormone that stores carbohydrates,lipids n proteins in body is:
a)growth hormone (correct)
b)thyroid hormone
c)cortisol
d)insulin
e)PTH

question no 61:turners syndrome,most definitive diagnostic test:
a)presence of barr body
b)chromosomal analysis (correct)

question no 62:14 yrs girl has primary amenorrhea,normal breast development,secondary sexual characteristics normal,short blind ending vagina,slight pubic hair,usg revealed absent uterus,underlying cause:
a)congenital adrenal hyperplasia (correct)
b)chromosomal defect
c)failure f development of genital tubercle
d)defect of paramesonephric duct

question no 65:repeated blood transfusions (AB n Rh compatible) in an individual will lead to:
a)haemochromatosis of liver
b)hypocalcemia (correct)
c)haemoglobinuria
d)transfusion reaction

question no 66: Rh incompatibility occurs in a mother if she has:
a)Rh negative fetus (correct)
b)Rh positive fetus

question no 67:the best way to prevent Rh isoimmunisation in a woman who has given birth to Rh positive fetus:
a)blood transfusion
b)platelet transfusion
c)serum
d)Rh immunoglobulins (correct)
e)

question no 68:a person brought in er had burns,he was admitted later on he developed fever and bleeding diathesis,he is suffering from:
a)DIC (correct)
b)hepatorenal failure
c)hepatic failure of prothrombin production
d)renal failure

question no 69:following is not a cause of DIC:
a)pre eclamsia
b)amniotic fluid embolism
c)cervical carcinoma
d)abruptio placentae (correct)
e)PPH

FCPS PAST PAPERS POOL PART-1 OB/GYNA with answers

.Lymph drainage of perianal skin is from one of the following

A.internal iliac nodes
B.external iliac nodes
C.lateral group of horizontal superficail nodes
D.medial group of horizontal superficail nodes(correct)
E.vertical group of nodes

4.Lesser omentum
A.connects with deudenum
B.forms falciform ligament b
C.related to left adrenal gland
D.related to left kidney
E.related to rt kidney

9.Which of the following is not a feature of shock
A.decrease cardiac output
B.muslce weakness
C.increased urine output(correct)
D.tachycardia
E.restlessness


39.Which is most imp indicator of muscle protein loss

A.urea a
B.ammonia
C.none of above
D.uric acid
E. ?

40.wat is true about DNA

A.Double stranded in which two strands are antiparallel to each other
B. a single turn of DNA is 2nm
C.Can exist as helix in single stranded form as well as in Double stranded form
D.
E.

41.A normal looking Girl came to you with primary amenorrhea,on examination she was having short blind vagina wid normal vulva, scanty pubic n axillary hairs,normal breasts,absent uterus,her karyotype is?

A.46XY (correct)
B.47XY
C.47XXY
D.45XO

45.A pt came to you with primary amenorrhoea, webbed neck, short stature diagnosed as Turner syndrome, wht is her karyotype?

A.45XO (correct)
B.46XY
C.47XXY
D.46XX
E.......
10.supply of sphincter urethra comes from
A.ilioinguinal nerver
B.pelvis splanchnic nerves
C.sacral splanchnic nerves
D.autonomic nerves
E.Pudendal nerves (correct)

15.Thymus develops from

A.ectoderm of 2nd brachial pouch
B.endoderm of 2nd brachial pouch
C.ectoderm of 3rd brachial pouch
D.endoderm of 3rd brachial pouch (correct)
E.mesoderm of 3rd brachial pouch

16.A pt presented with an opening in the ant wall of sternocledomastoid muscle with pussy discharge coming out of it.this problem occured due to abnormality in one of the following

A.1st pharyngeal cleft
B.2nd pharyngeal cleft
C.1st pharyngeal pouch
D.2nd pharyngeal pouch (correct)
E.3rd pharyngeal cleft

17.Diaphragmatic hernia occurs due to

A. absence of pleuro-pericardial membrane
B. absence of septum transversum
C. absence of pleuro-peritoneal membrane(correct)
D. absence of central tendon
E. absence of arch?

18.Erythropoises occurs in middle trimester mainly from

A.liver
B.kidney
C.spleen(correct)
D.bone marrow
E.both liver n kidney

19.Superior parathyroid gland develops from

A.1st brachial arch
B.2nd brachial arch
C.3rd brachial pouch
D.4th brachial pouch (correct),
E.3rd brachial cleft

20.In cervical carcinoma,wat is the lymph drainage

A.internal iliac
B.external iliac
C.both internal n external iliac (correct)
D.superficial inguinal
E.deep inguinal

21.RT coronary artery

A.arises from anterior aortic sinus (correct),
B.enters the rt heart border after passing through sulcus on posterir surface
C. supplies both ventricles
D. supplies S.A node
E. supplies left atrium

25.Most imp hormone involve in gluconeogenesis is

A.growth hormone
B.insulin
C.thyroxine
D.cortisol (correct)
E. Glycogen

26.A pt presented with rt iliac fossa pain. he is diagnosed to have acute appendicitis on flexing his rt thigh inward n medially he cries with pain.wat is the type of appendicitis he has

A.paracolic
B.retrocolic
C.pelvic (correct)
D.paracecal
E.subcecal

1.Apoptosis is inhibited by

A.capsases activation (correct),
B. Bcl-2 inhibition
C. Activation of P-53
D. apoptosis related protein ??
E…


32.Most potent chemotactic factor is

A.C3a
B.C5a (correct)
C.IL-1
D.viruses
E.arachdonic acid metabolites

33.what is the most appropiate answer

A.C3a is opsonin
B.C5a is potent anaphylactic
C.C5a is a potent chemotactic agent (correct)
D.C3b n C5a are anaphylactant
E.......

34.Low serum complement seen in which condition

A.SLE
B.ankylosing spondiolitis
C.rheumatid arthritis
D.multiple sclerosis
E.dermatomiositis

35.Pt presented with photosensitivity,rash n joint pain.which is the daignostic in this case

A.anti SS antibodies
B.anti DS antibodies (correct)
C.RA
D.anticentromere antibodies
E.antimitochondrial antibodies

47.Which of the following has highest cholesterol content

A. Chylomicron remnants
B. LDL(correct),
C. VLDL
D. HDL
E. IDL

49.Which drug do not cause gynaecomasia?

A.Digoxin (correct)
B.Griseofulvin
C.Cimetidine
D.Androgen
E.spironolactone

50.Which is the best option in identification of Turner's syndrome

A.ultrasound
B.barr bodies
C.karyotyping (correct)
D.Ct scan
E.

53.End product of purine metabolism is

A.urea and water
B.uric acid(correct),
C.glucose
D.water and ammonia
E.ammonia

54.Erythropoitin is secreted from

A.liver cell
B.peritubular interstitium of kidney(correct),
C.macula densa of kidney
D...
E...

55.ESR is decreased with

A.temprature
B.albumin is decreased (correct)
C.globulin is decreased
D.decreased blood volume
E.decreased plasma volume

56.RBC's are

A.biconvex
B.nonfragile
C.have nucleus
D.largest of all cells
E.have glycolytic enzyme activity (correct),

59.Commonest Cause of PDA

A.drugs
B.toxin
C.permaturity (correct)
D.congenital abnormality
E..infection

60.Most common Cause of metaplasia is

A.ultrasonography
B.chronic irritation (correct),
C.infections
D.radiations
E.

61.Wat is the differentiating point of hyperplasia fron hypertrophy

A.increase in size of cell
B.increase in size n no of cell
C.is reversible
D.is premalignant ???????
E.involve viscera only

62.Dysplasia is seen mainly seen in

A.bone marrow (correct)
B.cartilage
C.epithelia
D.fibrous tissue
E.deep tissue

63.Wat is the differentiating feature of hyperplasia from benign tumor

A.increase in no of cell (correct),
B.are the same as parental cell
C.cytplasm n nucleus ratio is derranged
D....
E...

64.What is the most true among following

A.mRNA has a codon
B.mRNA has anticodon
C.rRNA is most abundant n involve as ribosomes on endoplasmic reticulum(correct)
D.tRNA is largest
E.tRNA involve in protein synthesis

65.deficiency of THAIMINE(B1)causes all except

A.cardiac anomalies
B.muscle wasting (correct)
C.defects in alcohlics
D.subacute degenaration of spinal cord
E.cns abnormalities in alcoholics

66.AIDS is associated with all of the following except

A.kaposi sarcoma
B.lymphoid tumors
C.primary tumor of brain (correct),
D.STD
E.leukemia

67.VITAMIN involve in collagen synthesis is

A.ascorbic acid(correct),
B.biotin
C.thiamine
D.pyridoxine
E.niacin

68.Neural tube defects occur due to deficiency of

A.iron
B.folic acid (correct),
C......

69.Iron is stored in the form of

A.ferritin
B.transferrin(correct),
C.apoferritin

70.CO2 is tranported in lungs through
A.simple diffusion (correct),
B.fasilitated diffusion
C.active transport
D.carriar proteins
E.channels

71.HSV is associated with
A.CA cervix
B.CA vulva
C.vaginal wart
D.vulval papules?

74.Optic groove appears on left side of forebrain on day
A.12
B.22 (correct)
C.32
D.42
E.52

75.Germen measles causes which abnormality. What is the most appropriate answer
A.congenital cataract (correct),
B.congenital deafness?
C,cardiac anomalies?
D.limb deformaties
E........

76.A newborn with anterior abd wall swelling and umbilical cord attached to it.wat is the condition called
A.omphalocele (correct)
B.gastroschiaosis
C.umblical hernia
D.omentocele
E.intestinal malformation

77.PT has exophthalmos n his T3 n T4 are increased.wat is the option for treatment of exophthalmos
A.drugs blocking the action of T4?
B.direct suppresion of T4 (correct)
C.hypophysectomy
D.administration of testosterone
E.partial parathyroidectomy

78.Treatment of diabetes in pregnant lady is
A.sulphonyl urea
B.insulin (correct),
C.biguanides
D.suphonylurea n biguanides
E...

83.side effect of streptomycin
A.impairment of hearing
B.
C.
D.
E.

84.Which of the following is opportunistic organism
A.E coli?
B.salmonella
C.shigella
D.klebsella
E.chlostridium (correct)

85.Psudomembranous colitis is caused by
A.Cl.difficile (correct)
B.Cl.tetani
C.CL perfiranges
D.Cl.botulinum
E..

86.About aspargillosis which is not correct
A.is caused by fungal
B.usually seen in preexisting lung diseases?
C.is caused by long term antibiotic use.
D...
E........

97.Pt has a history of infection 4 weeks back.now he developed same pain again.wat type of cell will present at this time of disease
A.neutophils (correct),
B.eosinophils
C.lymphocytes
D.basophils
E.mast cells

98.What is the drug of choice for DVT in ist trimester of pregnancy
A.aspirin
B.heparin (correct)
C.warfarin
D.indomethacin
E.mefanamic acid

102.Type 2 hypersensitivity reaction includes
A.erythroblastis fetalis
all other options were wrong

103.Transfusion reaction will not occur in a pt if we transfuse the
A.group A with O
B.group A with AB (correct)
C.group AB with A
D.group B with O
E.group O with B

104.Which of the following does not cross placenta easily
A.glucose
B.amino acid
C.Ig G
D.Ig M (correct),
E.CO2

105.Which drug is used in the treatment of hirsutism
A.antiandrogens (correct)
B.estrogen
C.testosterone
D.cryptoterone acetate
E.diacrone

106.Most common organism involved in infection caused by IUCD
A.actinomycosis
B.E coli
C.bacteroide
D.gonorrhea
E.trichomonas

107.ECF differ from ICF in
A.high conc. of Na n Ca (correct)
B.high K low Na
C.low Na n Ca high K
D. high conc. Of K & Ca with low Na
E.Volume …

108.diffrence of smooth n skeletal muscle
A.Ca-Calmodulin complex is present in smooth muscles

109.In partial mole,karyotype is
A.46XX
B.45XO
C.47XX
D.47XY
E.69XXY (correct)

110.If carbohydrate is separated from proteins wat will be the fate
A.aggregation n precipitation (correct)
B.eliminate fron kidney?
C.increase uptake int muscle

111.Lysozome contains
A.glucose
B.Ca
C.Na
D. hydrolases (correct)
E.alkaline phosphate

122. Following is Autosomal Dominant Disorder
a) Phenylketonuria
b) Hereditary Spherocytosis (correct)
c) Hemophilia A
d) Cystic Fibrosa

FCPS PAST PAPERS PAPER -1 (Medicine, Surgery, Radiology, OB/GYN)

FCPS PAST PAPERS PAPER -1 (Medicine, Surgery, Radiology, OB/GYN)

1-The muscle involved in unlocking of knee joint is:
Poplitus(correct),,Rectus femoris,Plantaris.Soleus

2-Adult derivative of notochord is:
Nucleus pulposis(correct),,Annulus fibrosis,Vertebre,Vertebral canal

3-Vertebrae is derived from:
Myotome,Sclerotome,Dermatome,Ectoderm,Endoderm

4-Memory centre is located in:
Insula,Parietal lobe,Temporal lobe,Frontal lobe

5-The most commom type of salivary gland tumor is:
Salivary glands,Parotids(correct),,Submandibular,Sublingual

6-Distruction of Ant.pituatary gland causes dec.functioning of
Glom.fasiculata,Glom.granulosa,Medulla,Adrenal cortex(correct)

7-Least chances of renal stones is associated with:
Hyperlipidemia(correct),,HyperPTH,Hyper vit.D,Infections,Hyperurecimia

8-Nerve supply to extensors of arm is supplied by:
Lat.cord,Medial cord(correct),Post cord,Lateral and medial cord

9-Nerve injured in Ant.disloacation of Shoulder joint is:
Musculocutaneous,Axillary(correct),Median,Radial,Suprascapular

10-Damage to scaphoid bone causes injury to:
Radial A. (correct),Ulnar A.Brachial A,Ant.interosseous A.

11-Common site of fracture at rib is:
Angle(correct),,Shaft,Costochondral joint,tubercle

12-Following is not a tumor marker:
PLAP,CEA,bHCG,AFP,Acid Phospatase(correct),

13-Most common histology found in lung tumors is:
Squamous(correct),,AdenoCA,Mixed,Small CA

14-Patients comes with Rt.deviation of tongue,Dec.sense of touch and vibartions,the artery
commonly involved in brain is:
PICA,AICA,Ant.Spinal(correct),,Post.Cerbral,Sup.Cerbral A.

15-Soldier comes with heavy bleeding,the ideal fluid replacement would be:
Packed RBCs,Crystallines,Colloids,Whole blood for 3 days(correct),,Whole blood for 18 days

16-Maxillary sinus opens into:
Sup.meatus,Inferior meatus (correct),Middle meatus,Nasal septum

17-Patient having increased levels of IgE,most probably suffering from:
Bacterial inf,Viral inf,Parasitic inf(correct),,Fungal inf.

18-Autospy done on the patient having CRF,most prabable finding would be:
Hypertrophied PTH gland(correct),,Hypertrophied adrenals,Hypertrophied pituatry,Hypertrophied thyroids

19-Excitation of post.pituatary would cause:
Sweeting,Stress,Shivering,Hunger pangs

20-Foetal period starts after which week:
11th.8th(correct),12th,16th,21st

21-Patient having dec levels of factorII,VII,IX,X,most likely have:
Dec.antithrombin III,Dec Vit.K(correct)

22-Young pt.having Hb:6.0,TLC:3.5/ul,Plts:1lac is suffering from:
Iron def.,Folate def.,Vit B12 def.,Aplastic anaemia(correct)

23-Nucleus ambigus supplies to:
Teeths,Jaw,Tongue,Lyrnx(correct)

24-Olfactory area is present in:
Ant.perforating material,Occipital lobe,Inf.temporal gyrus,Parietal lobe

25-Patient with injury to left 8th cervical segment of spinal cord will not show following sign:
Dec sense of position,vibration below lesion on same side,Extensor plantar on left side,Dec
power of muscles below the leson on same side,Dec sense of pain and temprature below the
lesion on same side. (correct)

26-Feature of Rickets and Osteomalacia is:
Inc.Vit D,Inc.Ca,Inc PTH,Dec.bone density(correct)

27-Most common site of malignancy in pts suffering from nuclear outbreak
Haematopoitic,Thyroid,Lung(correct),Breast,Bones

28-Virus acts on cells by:
Damaging cell membrane,Damaging nuclear membrane,Invloving protien synthesis(correct),

29-Most sensitive cells to hypoxia are:
RBCs,Neurons(correct),,Nephrons,WBCs,Platelets

30-Myelin in CNS is formed by:
Microglia,Schawan cells,Oligodendrocytes(correct),,Astrocytes

31-Feature not associated with irreversible cell injury is:
Mylein figures(correct),,Karyolysis,Karyorrehxis,Mitochondrial shrinkage

32-Ureter isnot constricted at:
Pelvic brim,at PUJ,Psoas muscle(correct),Openning at bladder

33-Common relation of ureter is:
Ant.to gonadal vessels and post.to iliac A(correct).,Post.to gonadal vessels and ant.to iliac A.

34-Patient feels pain after she underwent surgery for fibroids,structure injured would be:
Ureters,Ovaries,Fallopian tubes(correct)

35-Least common site for ectopic pregnancy would be at:
Ovaries(correct),Pouch of douglus,Greater omentum,Fallopian tubes,Cervix

36-Gall bladder is supplied by:
Cystic A and Left gastrcA.,Cystic A and Left Hepatic,Only Cystic A. (correct),

37-Venous drainage of urinary bladder is into:
Internal Iliac V(correct),.,External Iliac V.,Commom Iliac V.,Internal and external V.

38-Hypothyroid patient in on thyroxine,best marker to monitor his thyroid status is:
T3,T4,TSH,T3 and T4,T3,T4 and TSH levels(correct)

39-Patient having hyperplastic bone marrow,anaemia,Inc Platelets is probably suffering from:
Aplastic anaemia,Megaloblastic,Myelofibrosis,Leaukemia(correct)

40-Stimulus for Eryrthropoiten secreation is:
Hypoxia

41-Erytropoiten is always raised in:
Polycythemia vera,Primary polycythemia(correct),Sec.polycythemia
(Polycythemia vera is only type of polycythemia in which there is Dec. EPO.
42-Following vein would be dilated in portal HTN:
Inf.epigastric,Gonadal,Renal vein,Left colic V(correct).

43-Patient after gastrectomy is on VitB12,the cells lost in the gastrectomy are:
Mucous,parietal(correct),cheif,Goblet cells.

44-Fat tags attached to the walls of large intestine are known as:
Taenia coli(correct),,Hausstral folds,Appendeces epiplocea

45-Bile salts are reabsorbed from the:
Duedenum.Jejunum,Ileum(correct),,Colon,Rectum

46-Following is an autosomal dominant disease is:
Cystic fibrosis,Hereditary spherocytosis(correct),Thalasemia,Sickle cell anaemia

47-Type of defect in Heredetary spherocytosis is:
Enzymatic defect,Structural defect(correct),

48-Patient with jaundice,anaemia,high reticulocyte count is having:
Folate def.,Iron def.,VitB12 def.,Hemolytic anaemia(correct)

49-Ejaculatory duct opens into:
Ureter,Ureteric crest,Bladder,Prostatic urethera(correct)

50-Terminal part of CBD in relation to pancrease is:
Embeded into pancrease,lie posteriorly,lie anteriorly

51-Following would cause massive infarction and destruction leading to patient death:
Thrombus,Embolus,Fatty thrombus,Thromboembolism(correct)


52- 22yrs female pt.with 13th week of pregnancy after having crush fractures in RSA dies after 3
days,most likely cause of death is:
Amniotic fluid embolism,Air embolism,Fat embolism(correct)

53-Patient with old history of adenoCA of colon is operated for polypectomy,on histologic evaluation
pathologist labelled it as benign growth with no chances into malignant transformation,it would be:
FAP,Villous adenoma,Tubular adenoma,Tubulovillous(correct),Metaplastic polyp

54-Tyrosine derivateve doesnt include:
TSH,Adrenaline,Nor adrnaline,Prolactin,Dopamine

55-Precursor of steriod hormone is:
Progesteron,Pregnenolone,Cholesterol(correct)

56-Capsule of liver in known as:
Disse,Glissons(correct),

57-Most common site of thyroglossal cyst is:
Suprahyoid,Infrahyoid,Hyoid(correct)

58-Wasting of thenar eminence,loss of sensation over thumb and index finger,nerve involved is:
Median(correct),,Axillary,Radial,Ulnar

59-The base of urinary bladder is made by:
Post surface(correct),Ant surface,Inferolateral,Superior surface

60-Shock without having vasodilation is likely associated with:
Burns,Gram positive inf,Gram negative inf.,Superantigen infection

61-Following doesnt form the layer of inguinal canal:
External oblique,internal oblique,transversus,rectus abdominus. (correct)

62-Femoral sheath is formed by:
Fascia transversalis,Fasica trans.and Fasica iliaca(correct),,External oblique,Internal oblique

63-Patient with bone pains having normal Ca,inc Alk.phosp.most likely suffering from:
Pagets,HyperPTH,Hyper VitD,Bone mets(correct),Osteomalacia

64-External spermatic fasica is formed by:
External oblique(correct),Internal oblique,Cremastric muscle,Transversus muslce

65-In Turner syndrome,the genotype would be:
XX,XO(correct),XXY,XY,XYY

66-Organ having least chances of infarction:
Lungs,Heart,Kidneys,Liver(correct),Spleen

67-Type of necrosis in brain infarction:
Couglative,Liquefactive(correct),,Fat,Traumatic

68-Pt.with cervical lymphadenopaty,+ve AFB sputum,sign to look for TB on microscopy is:
Chronic inflammation.caseous necrosis(correct),lymphocyctic infiltartion

69-Thyroid gland moves with swallowing because it is enclosed in:
Pretracheal fasica(correct),Investing fasica,Vertebral fascia,Carotid fascia

70-Neurovascular bundle in chest wall lies between:
External and Internal intercostal mucles,Internal and Innermost layers(correct), Innermost and enothoracic
fasica,Beneath Endothoracic fascia

71-Fibrous pericardium and medicatinal pleura is supplied by:
Vagus N.,Intercostal N.,Phrenic N(correct),.,Autonomic N.

72-Man is suffering from testicular ca,the lympahtic drainage of testicle is into:
Para aortic L.nodes(correct),,Pre aortic,Superficial inguinal,Internal iliac,External iliac L.nodes

73-In exercise,the venous blood returns to the heart by:
Muscle pump in calves

74-Circulation in heart is maintained by:
Symphthatic sys,Parasymphthetic sys,Local mechanism(correct)

75-Fisherman is presented with gingival bleeding and ecchymosis,he is suffering from:
Folate def.,Iron def.,Vit B12 def,Vit C def(correct)

76-Facor affecting collagen synthesis during healing is:
Infections(correct),,Vit C,Foriegn body

77-Mesothelioma is associated with:
Vinyl chloride,Silica,Asbestos(correct),,Copper dust,Carbon

78-Child having meckels diverticulum is having bleeding per rectum,it is due to:
Volvulus,Interssusption,Ectopic gastric tissue(correct)

79-During lactation,Amenorrhea is due to:
Inhibition of LH and FSH thru Prolactin(correct),,Dec.prolactin,Dec oestrogen

80-Presence of pancratic tissue in gastric mucosa is termed as:
Hamartoma,Metaplasaia(correct),Neoplasia,Choristoma,Dysplasia

81-Main difference between primary and secondary intention wound healing is:
Granulation tissue,Wound contraction(correct),Inflammatory cells

82-Child with sore throat after 3 weeks developed oedema,haemturia,cause would be:
Minimal change disease.post streptococcal GN(correct),IgA nephropathy,Focal segmental GS.

83-Microscopic feature of polyarterites nodusa is:
Granuloma,Fibrinoid necrosis(correct)

84-Angiotension II exerts its effects by activating:
Renin,Angiotension I,Aldosterone(correct),ANF

85-Tumor compressing optic chiasma would cause:
Binasal haemianopia,Bilateral haemianopia,Bitemporal haemianopia(correct),,Complete blindness

86-Growth hormone is maximally rasies in:
Sleeping(correct),,Excercise,Hunger,Anxiety

OBS/GYNA FCPS PART-1 NEW POOL (paper 1 & 2)


Lamellar bodies are found in:
A- Clara cells
B- Type 1 alveolar cells
C- Type 2 alveolar cells
D- Macrophages

Ans C
surfactant contained within the lamellar which is released via exocytosis.


Best method to detect malignancy
A- Cytology
B- Histopathology
C- Immunochemical

Ans B


HIV diagnosis is through
A- western blot
B- Elisa
C- CD count >300

Ans A
Vitamin D causes:
A- increased renal Ca re-absorption
B- increased serum phosphate levels

Ans A

Antithrombin
A- Has a structure similar to thrombin
B- is a valine protease inhibitor
C- When binds to protease action is increased by heparin

Ans C
it is a serine protease inhibitor. In antithrombin III deficiency heparin's anticoagulant effect is decreased.


Head injury causes paraplegia with no sensory loss which are is involved?
A- Cerebral cortex
B- Spinal cord at T10 level
C- Pons
D- Medulla

Ans A
Tricky! I guess what they are referring to is the involvement of motor cortex!!
ADH
A- Has double size compared to oxytocin
B- Released on decreased osmolarity
C- Is transported down attached to neurohypophysin

Ans C

HIV infection in female will cause
A- vulvar cancer
B- endometrial cancer
C- cervical CA
D- leukemia

Ans C
Following is not a sign of irreversible cell injury:
A- Karryorhexis
B- Karyolysis
C- Mitochondrial shrinkage
D- myelin figures


Ans D
still reversible at this stage, but if the conditions don't improve irreversible injury will develop


Type of necrosis in omentum:
A- liquefactive
B- coagulative
C- gangernous
D- fat necrosis

Ans D
A kidney transplant immediately turns blue and is rejected within minutes:
A- type 4 sensitivity
B- Ab. mediated
C- Cytotoxic rejection
D- GVHD

Ans B
a question of hyper-acute rejection.

Which of the following causes viral mediated host DNA transfer:
A- Transduction
B- Transformation
C- Conjugation

Ans A

Which of the following most probably has an animal reservoir?
A- influenza
B- measles
C- smallpox

Ans A
Vitamin K deficiency following is unusual:
A- coumarin therapy
B- Bile duct obstruction
C- red meat ingestion
D- in newborn due to lack of transport through placenta

Ans C


Liquid dysphagia is due to:
A- hypoglossal nerve palsy
B- plummer-vinson syndrome
C- palatal palsy
D- neuromuscular incoordiantion

Ans D
Heinz bodies are found in
A- autoimmune hemolytic anemia
B- G-6-P-D deficiency
C- hereditary spherocytosis
D- post splenectomy

Ans B


Hemolytic anemia is a feature of
A- acute myeloid anemia
B- cirrhosis
C- mycoplasma
D- SLE

Ans A
All the others.
Necrosis in omentum is
A- fat necrosis
B- caseous necrosis
C- gangrenous necrosis
D- fibrinoid necrosis

Ans A


When a question is asked from a student during lecture, he becomes red faced. This is due to
A- active congestion
B- active hyperemia
C- passive hyperemia
D- parasympathetic stimulation

Ans B


Testosterone is secreted by interstitial cells of leydig it
A- increases hair growth on scalp
B- increase protein synthesis in bones and muscles
C- inhibits secretions of sebaceous glands
D- it is 17-ketosteroid
E- is converted into androsterone in target cells

Ans B
Para amino hippuric acid is used to determine
A- GFR
B- renal blood flow
C- renal plasma flow
D- renal creatinine clearance

Ans C


Earliest side-effects of aspirin over-dose
A- tinnitus, vertigo
B- vomiting
C- seizures

Ans B


A scenario of child having serum cholesterol of 600mg/dl, his father, mother and elder brother are all having this high level of cholesterol in their blood.What gene mutation is responsible for this condition?
A- apoE
B- HDL receptors
C- LDL receptors
D- basal chylomicron transport proteins
E- VLDL receptor

Ans C
A scenario of child having Hb 6gm/dl, WBC is 5200 and platelets 150000/mm3. his elder brother also has a history of repetitive blood transfusions. ow you come to a diagnosis for this child?
A- CBC
B- Immunocytochemistry
C- Electrophoresis
D- Sickling test
E- Fragile test

Ans C


Diagnosis of a tumor is made by
A- cytological examination
B- histopathological examination
C- immunochemical assay
D- enzyme assay
E- tumor marker

Ans B
80% of oxygen is extracted by which organ?
A- heart
B- skeletal muscle
C- lungs
D- brain

Ans A


A scenario of patient having Factor VIII deficiency. The genes responsible for this disease are at chromosome?
A- 21
B- 14
C- 11
D- X
E- Y

Ans D
A surgical patient is transfused 3-5 units of blood before and after surgery. He develops abnormal ECG changes not consistent with ischemia. What is the cause of the ECG changes?
A- metabolic alkalosis
B- hypercalcemia
C- hyperkalemia

Ans C


Glucagon increases glycogenolysis in liver while ACTH does not due to?
A- ACTH increase plasma glucose
B- glucagon has different guanyl cyclase than ACTH
C- liver has different receptors for ACTH
D- ACTH is readily degraded by the liver enzymes

Ans A
ACTH via cortisol causes gluconeogenesis.


If celiac trunk is blocked which of the following will not suffer from ischemic injury?
A- liver
B- pancreas
C- spleen
D- gall bladder

Ans B
Head of pancreas is in front of of aorta
A- inferior vena cava
B- celiac trunk
C- aorta
D- renal arteries
E- superior mesenteric artery

Ans A


Carcinoma breast metastasizes to
A- lung
B- bone
C- brain
D- kidney
E- liver

Ans B

Lung contains metastatic tumor, histopathology reveals clear cells. Most commonly this tumor has metastasized from
A- kidney
B- liver
C- bone
D- adrenal
E- breast

Ans A


Regarding salivation
A- caused by salivatory nucleus of medulla/pons
B- increases during sleep
C- inhibited by pilocarpine
D- increases in response to Beta-adrenergic stimulation

Ans D


Beta adrenergic stimulation causes
A- mydriasis
B- increased blood flow to skeletal muscles
C- increase peristalsis

Ans B


Which organism is found in urine sample
A- paragonimus westermani
B- wuchereria bancrofti
C- schistosoma hematobium

Ans C


Which pus forming organism in lung also causes meningitis?
A- streptococci
B- staphylococci
C- bacteriodes
D- pneumococci
E- psuedomonas

Ans D


Common premalignant condition of mouth in adults
A- lichen planus
B- hyperkeratosis

Ans A


Most lethal premalignant condition is
A- lichen planus
B- leukoplakia
C- erythroplakia
D- solar keratosis

Ans C
is there anyone preparing for FCPS 1 in gyne obs or any other speciality in june or oct?
let's discuss the past papers in the forum one by one.
1.acute inflammation causes

a.Transudate
b.Exudate
c.Inc protein content
ans:b


2.verbal communication skill depends on

a.Speak fluently
b.Education nd research training
c.Presentation
d.Good vocabulary
ans:c

3.which artey lie in true pelvis and crosses frm med to lat side of pelvis

a.Sup rectal art
b.Middle rectal art
c.Inf rect art
d.Internal iliac
ans:d


4.urogenital diagphram inserted on

a.Ishchial tuberosity
b.Ischial spine
c.Sub pubic arch
d.Ischial ramus
ans:c

5.which prevent heat loss
a.Shivering
b.Vasoconstriction
ans:i think both prevents so wht's the answer?

6.which drug causing minimal GI side effects

a.Paracetamol
b.Aspirin
c.Ibuprofen
d.Ketorolac
e.Diclofenac
ans:c
on Saturday •LikeUnlike • •UnsubscribeSubscribe


7.yellow fever inf occurs ,.,.,..,.,.,.,

endemic
sporadiac
pandemic
epidemic
ans:this is so confusing i think yellow fever comes in all these categories so wht's the answer?

8.last mediator of endotoxic shock...,..,

IL1
IL6
TNF ALPHA
LIPOPOLYSACHARRIE
ans:don't know any body knows the answer?

9.germ cells derived from.,

a-ectoderm
b-endoderm
c-mesoderm
d-neural crest cell
ans:c


10.wall of inguinal canal Not formed by?

a.ext oblique apponeurosis
b.lacunar lig
c.internal oblique
d.conjoint tendon
transversalis fascia
ans:c
After renal transplant which malignancy is most common?

Ans Common malignancies include:
1. Skin cancer
2. Melanoma
3. Kaposi
4. Post transplantation lymphoproliferative disorders



Commonest site of lumbar puncture:
A- L4-L5
B- L2-L3
C- S1-S2


Ans A


Which is not punctured during lumbar puncture:
A- Duramater
B- Ligamentum flavum
C- Longitudinal spinal ligament

Ans C

HLA-DR4 is associated with:
A- Rheumatoid arthritis
B- SLE
C- Scleroderma

Ans A


Which causes dysphagia
A- Ankylosing spondylitis
B- Scleroderma
C- Dermatomyositis

Ans B


Which is the tributary of portal vein:
A- Superior portal vein
B- Inferior portal vein

Ans A

Which is associated with cancer?
A- Methyl alcohol
B- Propyl alcohol

Ans B

Tunica vaginalis is derived from
A- Transversalis muscle
B- Transversalis fascia
C- Peritoneum
D- Internal oblique

Ans C

Turner syndrome has the karyotype:
A- XO
B- XX
C- XXY
D- XXX

Ans A

Cyanosis is due to
A- Increased deoxy hemoglobin in blood
B- Decreased PO2 in blood

Ans A

A lady suffers an injury in RTA and is unable to flex both her leg and thigh. The muscle most likely affected is:
A- Biceps femoris
B- Rectus femoris
C- Semimembranosus
D- Sartorius
E- Semitendinosus

Ans D
In Type 4 hypersensitivity mainly cells are:
A- T cells and macrophages
B- B cells and macrophages
C- IgE and T cells
D- B+T cells
E- None

Ans A


Malignant tertian malaria is caused by:
A- P. falciparum
B- P. malariae
C- P. knowlasea
D- P. ovale
E- P. vivax

Ans A


In duodenal perforation the artery involves:
A- Gastroduodenal
B- Left gastric
C- Splenic
D- Right gastric

Ans A


Lesion of mammillary body causes:
A- Fear and rage
B- Hyper sexuality
C- Loss of recent memory
D- Hyperphagia

Ans C


Heat loss of body is controlled by:
A- Lat. hypothalamus
B- Preoptic hypothalamus
C- Post. hypothalamus
D- Thalamus
E- Hippocampus

Ans B

Heart muscle functions as a syncytium because of:
A- Sarcomere
B- SER
C- Gap junction
D- Tight junction
E- Intercalated disk

Ans E
Central venous pressure:
A- Increases in hemorrhage
B- Decreases in gram negative septicemia
C- Decreases in heart failure

Ans B

Stroke volume:
A- Cardiac output depends on it
B- Heart rate determines it
C- Increases in hemorrhage
D- Independent of venous return


Ans A


Sacralization:
A- Union of 1st sacral vertebra with 5th lumbar
B- Union of 5th lumbar vertebra with 1st sacral
C- Fusion of all sacral vertebra to form sacrum
D- Flexion at sacrum

Ans B



What change occurs form lying to standing position?
A- Increase in venous return
B- Increase in arterial pressure
C- Sweating
D- Cutaneous vasoconstriction


Ans B



Causes of gas gangrene:
A- C. botulinum
B- C.C. Ijungdahlii
C- Tetanus toxin
D- C. septicum

Ans D


C. perfringens acts by:
A- producing lecithinase
B- Depleting ATP
C- Causing hypoxia


Ans A


Which one of the following is not associated with dry gangrene?
A- DM
B- Thromboembolism
C- Bacterial endocarditis

Ans C

About leukocyte adhesion factor responsible:
A- integrin
B- selectin

Ans A


Diagnosed case of tuberculosis PPD or tuberculin test found to be negative:
A- ATT
B- Immunosuppressive
C- Steroid intake

Ans B

5.2.11

OBS / GYNAE FCPS PART 1 PAPER 1 AND 2 POOL (PAST PAPER COLLECTION WITH KEYS)


GYNAE OBS JUNE 2009
Gyn Obs JUNE 2009.


1-Etiology of cleft palate.

a-anticonvulsant
b-multiple preg
c-multifactorial
d-inc maternal age

2-left cleft palat dt malformation of.,

a-palatine shelves
b-maxillary prominence
c-med nasal prominence
d-lat nasal prominence
e-incisive foramen

3-germ cells derived from.,

a-ectoderm
b-endoderm
c-mesoderm
d-neural crest cell

4-karyotype of turner syndrom,,.......,,,,(smethng like dat exactly nt remembr)
a-45XO

5-which 1 contain granulosa cell,,

a-graffian follicle
b-,,,...,..,

6-in primary infertility,,diagnosis of failure of ovulation by ??

a-wid lower abdominal pain,,..,,
b-rise in basal body temp
c-middle of menstual cycle,,,
d-thick mucosa sec frm cx wch nt show fern pattern....

7-pentamer immunoglobulin is???

a-IgA
B-IgG
c-IgM*
d-IgD
e-IgE

8-Regarding entameba histplytica....

a-it is a nematode
b-it causes liver disease
c-occurs in both trophozoite nd cyst stage
d-it exist normally in intestine
e-..,.,.,.,.,

9-cornybacterium diphtheria..,.,

a-gm -ve bacilli
b-club shaped*
c-,,..,.,.

10-mode of cell injury by clostridium_____.....,,,,,???

a-ATP depletion.
b-.,.,.,.

11-shigella..

endotoxemia
neurotoxin
verotoxin
enterotoxin
exotoxin

12-corneal opacity caused by..

ethambutol
quinine
phenothiazine
chloroquine
.,,.,.,.,..,.....

13-drug inducer by p450 mechanism,..

cimitidine
ketokonazole
phenobarbitone
,.,.....,,,,,,,....
,.,.,.,.,.,.,.,.,.

14-last mediator of endotoxic shock...,..,

IL1
IL6
TNF ALPHA
LIPOPOLYSACHARRIDE

15-yellow fever inf occurs ,.,.,..,.,.,.,

endemic
sporadiac
pandemic
epidemic
.,.,.,.,..,.,.,

16-wall of inguinal canal Not formed by???????

ext oblique apponeurosis
lacunar lig
internal oblique
conjoint tendon
transversalis fascia


17-endocervical polyp in pregnancy,,,
Physiological
.,.,.,.,.,.,.,.,.,…

18-vaccination of mycobacterium T.B based on

Type I hypersensitivity
Type II
Type III
TYPE IV Cell mediated

19-after subdivided no of peoples into gps ,,sampling is taken a/c to age ….wat type of sampling it is
Random sampling (other options nt remember, ,,,stratified etc ,,,,,nt sure)

20-ultrasound..

4-8 weeks transabdominal u/s,, fetal heart sounds can detected
Cresenteric line seen w/h cnfirm pregnancy,,,,(smething like dat )


21-MRI
Frequencies,,,,,,,,,,,,
,nd methaemoglobin.,,,,,,,,,,, (these sort of alien words were written)

22-bone scan ,,,,,,
(name of sme disesases were written)

23-incidence,,,.,
No. of new cases in a population in a time given

24-pie chart

25-wch 1 is absent in muscles

Glucose 6 po4ase
Hexokinase


26-end product of glucose metabolism

Lactic acid
1molecule of pyruvic acid
2 molecule of pyruvic acid

27-hormone ass with glucose ,fats ,and protein storage

Insulin

28-fetal brain development

Growth hormone
Thyroid hormone

29-w/h of the following amino acid should be taken in diet

Alanine
Tyrosine
Glycine’glutamine

30-folllowing phagocytic cell present in circulation?

Monocyte
Macrophage
Lymphocyte
Neutrophils
31-morphine can be given in

Biliary cirrhosis
Terminal ill patient of cancer
Acute pancreatitis

32-functional layer of endometrium consists of

Stratum basal
Stratum compactum nd stratum spongiosum
Stratum basal nd compactum

33-maldevelopment of uro-rectal septum ,,

Rectovag fistula
Rectouterine fistula
Imperforate anus
Rectoperineal fistula

34-structure lying b/w labium majus nd minus

Urethral gland
Bulbourethral gland
Minor urethral gland

35-w/c 1 of following muscle not attached to tendon of perineal body

Ischiocavernosus
Ext urethral sphincter
Bulbospongiosus
Transverse perineal muscle

36-lymphoide nodule nt present in

Thymus
Spleen
Lymphnode



37-Notochord forms
Neural tube
Cns

38-epithelium of ovary
Simple cuboidal

39-pivot jt include

Atlanto occipital jt
Atlanto axial jt

40-sacroiliac jt
Fibrous .,.
Synovial.,.,
,,.,.,..,.,.,.,.

41-hamartoma is.,.,.,.,.,.,

42-an organ lined by stratified sq epi with inc cellularity, vascularity, nd elasticity,,tubular
glands in lamina propria
Vagina
Esophagus
Oropharynx
Duodenum

43-post relation of left ureter

Root of mesentry
Iliac vessel

44-outer lat quadrant of mamry gland drain into
Ant gp axillary lymph node

45-xerophthalmia dt
Vit A def

46-anti-oxidant
Vit E

47-w/h st having 3 layers of muscles
Fundus of stomach
Esophagus
48-fat embolism

49-regarding mammary tissues w/h is Not appropriate

Pect major, seratus ant,lie in deep relations
Can mov freely over retromammary space
Peau’d orange caused by lymphatic obstruction.,.,.,.,.,.,.,.,.
Puckering of skin dt subcut ductal infiltration.,.,.,.,.,.,.,.,.



50-lady died dt amniotic fluid embolism

During delivery
During c/s nd post partum
After post partum
During labour,delivery nd postpartum

51-pacinians corpuscles

Tactile touch
Fine touch
Temp
Pain
Vibration(I think it was nt in option nt sure)

52-H+ exchange in proximal tubules with

HCO3 reabsorption
K secretion

53-pat wid severly vomiting came wid breathlessness,,,PH=7.5,,HCO3=INC--------,,,PCO2=55

Metabolic alkalosis
Metabolic acidosis
Compensatory metabolic alkalosis

54-uterovag prolapse w/h lig is injured

Broad lig
Round lig
Transverse cx lig

55-alkaline po4ase rises in
Prostatic ca
BPH
Pancreatic ca

56-in ca mammary gland w/h drug is given in post menopausal lady

Tamoxifen
Clomiphen
.,.,.,.,.,.,.,.,.

57-in left shift O2 hbdissociation curve w/h 1 is increase
Fetal hb

58-in thalassemia there is

Poor hemoglobinization
Dyserythropoiesis
,.,.,.,.,.,.,,.,.,..


59-intravascular haemolysis diagnosed by

Reticulocytosis
Inc in hepatoglobulin binding
Inc TIBC
,.,.,.,.,.,.,.,.,……..

60-lady wid reddish cheeks, ,,arthritis,,,,,
SLE

61-in metaplasia w/h is inappropriate
Irreversible
.,,,,,,,,,,,,,.

62-acute inflammation

Transudate
Exudate
Inc protein content

63-w/h drug causing minimal GI side effects

Paracetamol
Aspirin
Ibuprofen
Ketorolac
Diclofenac

64-in 6 yrs old child meningitis occurs dt w/h organism

H-Infuenza
Ecoli
Niesseria

65-sepsis is diagnosed most appropriately by

Blood culture
Granulocyte <2000--->15000
H.R >100
HR>,.,.,…,

66-w/h is transmitted by mosquito
Yellow fever
..,.,.,.,.,.,.,.,.,

67-virus not associated with ca in human

HSV
HBV
HDV
HPV
Papova virus(papilloma vitus +polyoma virus)


68-w/h is more in human milk as compare to cow’s
Lactulose

69-gonococcus is best easily diagnosed by

Gram staining
Culture
ZN stain

70-w/h is completely covered by pretracheal fascia
Thyroid gland

71-is not an embryonic remanant

Lat umbilical ligament

72-oxytocin nd vasopressin originate frm

Hypothalamus
Neyrohypophysin
Autonomic ganglion

73-sucrose

Is a diasaccride
Metabolized into 2 glucose molecules
Reducing or non reducing (nt remember)

74-w/h of the following dis not ass with carcinoma

Lysosomal storage dis
.,.,.,.,.,.,.,.,.,.,.,.,.,.,.,.,.,.

75-rough endoplasmic reticulum
Protein synthesis

76-fragility of rbcs membrane

Spectrin
77-behavioural sciences .definition

78-verbal communication skill

Speak fluently
Education nd research training
Presentation
Good vocabulary

79-Not a function of beta blockers

-ve ionotropic
-ve chronotropic
.,.,.,.,.,.,,.
.,.,.,.,.,.,.,.,.,.

80- alpha blockers


81-centrally acting HTNsive

Clonidine
Methyldopa
.,.,.,.,.,.,.,.,.,.

82-w/h 1 decrease the muscle length

Golgi tendon organ
Muscle spindle
.,,,,,,,,,,,,,,,,,,,,,,,

83-cerebellar lesion,

Ataxia
Rigidity
Initiate muscle movement

84-pre-motor cortex lie in
Frontal lobe
85-coccygeal part of spinal cord lie at the level of w/h vertebra

C2
L2
S1
S2

86-lie in true pelvis nd crosses frm med to lat side of pelvis

Sup rectal art
Middle rectal art
Inf rect art
Internal iliac

87-inf fascia of urogenital diaghphram also called as

Perineal membrane

88-female pelvis also called as

Android
Anthropoid
Platypelloid
Mesatipellic

89-at crowning w/h muscle is cut during episotomy

Bulbospongiosis


90-lumbar puncture

L2 L3
L3 L4
L4 L5
S1 S2

91-Lactation,,,,,,
Hormones,,,wh stimulate

92-prolactin hormone
Stimulated by,.,.,.,.,.,.

93-pregnancy is diagnosed by
Gonadotrophin hormone

94-urogenital diagphram inserted on

Ishchial tuberosity
Ischial spine
Sub pubic arch
Ischial ramus

95-w/h muscle div submandibular gland into superficial nd deep part

Mylohyoid
Ant belly of diagastric

96-root of left lung
Azygous vein arched at root of left lung.

97-teratogenic

Alcohol
Caffeiene

98-carcinogenic
Methyl alcohol
Benzidine
99-pat with blood gp A should nt given

A+
A_
O
AB

100-. Regarding kidney ,w/h sequence is correct

renal art > arcuate>interlobar>interlobular>glomerular>efferent arteriole
renal>interlobular>interlobar>arcuate>efferent arteriole>glomerular
renal>glomerular>interlobar>interlobular>arcuate>afferent arteriole
renal>arcuate>interlobular>glomerular>interlobar>afferent arteriole


101-in muscle ca+2 release frm

sarcoplasmic reticulum
T-tubule
Sarcomere

102-sarcomere
b/w 2 z line

103-parasympathetic nerves r
3, 7, 9, 10

104-w/c is present in de cavity of cavernous sinus

Abducent nerv
Trochlear
Occulomotor

105-in haemophilia

PT INC
APTT INC
Inc bleeding time

106-gestational age estimated frm.,.,.,.,.

Abd circumference.,,.
Biparital diameter.,.,.,. wks
Crown rump length ,,, 8 weeks.

107-norepi + serotonin degraded by
MAO

108-at last trimester w/c uterus is sensitive to w/c hormone
Oxytocin

109-to prevent heat loss
Shivering
Vasoconstriction

110-inc K
Aldosterone sec


Gyn Obs June 25,2008

1..half life of dopamine is 5 min , how much time will it take to reach the steady state concentration.
a….16 sec
b….8 sec
c….10 sec

2..wt is the circulation time b/w brachial n cerebral circulation.
a….15 sec
b….30 sec
c….60 sec

3..wt is the most potent stimuli for aldosterone release.
a….ACTH
b….Renin

4..which statement regarding adrenals is wrong.
a….size at birth is equal to adult size

5..actions of glucagons are all except

6..actions of growth hormone r all except

7..which of the following are features of type 2 hypersensitivity reaction
a….IgG
b….Ig M
c….T cells
d….B cells

8..wt is the requirement for complete sterilization
a….heating at 151 degree C for …. Min

9..energy liberated on complete metabolism of protein is
a….4.1 k cal
b….5.3 k cal
c….9.1 k cal

10..energy liberated on fat metabolism is
a….9.1 kcal

11..thyroid isthmus is at level of
a….2nd 3rd tracheal ring
b….3rd 4th tracheal ring
c….5th 6th tracheal ring


12..blood supply of spinal cord is
a….vertebral arteries
b….spinal arteries
c….internal juglar arteries

13 end arteries are present in
a….spleen
b….brain
c….lymph node

14..derivative of endoderm is
a….mesentry

15..laryngeotracheal bud appears during
a….4th week
b…5th week
c….7th week

16..muscle cut on episiotomy is
a….superficial transverse perineal + bulbospongiosus

17..patient feels pain on defecation nerve involved is
a….pudendal nerve
b….inferior rectal nerve

18..almost 10 to 15 questions on NSAIDs and prostaglandins

19..one of the following regarding levator ani is true

20..epthelial lining of ureter is

21..muscular layer of ureter in lower third is

23..derivatives of pharyngeal arches (2 to 3 questions)

24..wt makes right border of heart
a….rt atrium
b….rt ventricle
c….rt atrium + rt ventricle

25..chemoatractant for neutrophil is,

26..factors affected by vitamin K deficiency



27..lymphatic drainage of cervix is
a….int n ext iliac nodes
b….int iliac nodes
c….ext iliac nodes

28..point of meeting of intrinsic n extrinsic pathway

29..oxyhemoglobin dissociation curve (2 questions)

30..simple sqamous epithelium is present in all except

31..protein content of CSF is

32..which has highest protein content
a….plasma
b….lymph
c….CSF

33..autosomal dominant n recessive disorders (2 questions like name was give n was asked wt is wt)

34..origin of ext oblique muscle is from….

35..boundaries of inguinal triangle, the site for direct inguinal hernia, are…

36..structure palpable through lateral vaginal wall is

37..cortisol causes decrease in
a….platelets
b….RBCs
c….lymphocytes
d….neutrophils

38 difference b/w benign n malignant tumors is( 2 almost similar questions)
a….invasion
b….metastasis
c….capsule



OBST&GYNEA FCPS-1(OCT 23 07) Q & A’s-

1.Which of the following does not form the wall of perineum

A.coccyx
B.pubic symphsis
C.ishcial tuberosity
D.sacrotuberous ligament
E.sacrospinous ligament

2.A pt presented with pain on defecation after severe constipation.pain in constipation occurs due to ?
A.autonomic nerves
B.Internal pudendal artey
C.pudendal nerve
D.perineal nerve
E.inguinal nerve

3.Lymph drainage of perianal skin is from one of the following

A.internal iliac nodes
B.external iliac nodes
C.lateral group of horizontal superficail nodes
D.medial group of horizontal superficail nodes
E.vertical group of nodes

4.Lesser omentum
A.connects with deudenum
B.forms falciform ligament
C.related to left adrenal gland
D.related to left kidney
E.related to rt kidney

5.A young girl of 25 yrs went with her husband on honey moon after one month of marriage.she suddenly experianced severe spasmotic pain in her rt iliac fossa.n presented in emergency in state of shock.most probable diagnosis is
A..acute appendicitis
B.renal colic
C.intestinal obstruction
D.ectopic Pregnancy
E.acute cholecystitis

6.which of the following structure is not palpable through vagina
A.cervix
B.rectum
C.ureteric stone
D.retrouterine pouch
E.vesicouterine pouch

7.The peritoneum covers the anterior abdominal wall which is continous with the perineum.it has fascia which has superior n inferior layers.the inferior fascial layer also called as
A.colle's fascia
B.scarpe fascia
C.perineal membrane
D.....fascia
E......fascia

8.If a known hypertensive pt took some diuretic n died in 6 hours.wat is the cause
A.acute renal failure
B.chronic renal failure
C.diuretic use
D.hypophosphatemia
E.hypokalemia ?

9.Which of the following is not a feature of shock
A.decrease cardiac output
B.muslce weakness
C.increased urine output
D.tachycardia
E.restlessness

10.supply of sphincter urethra comes from
A.ilioinguinal nerver
B.pelvis splanchnic nerves
C.sacral splanchnic nerves
D.autonomic nerves
E.Pudendal nerves

11.Rt ovarian vein drains the following

A.inferior vena cava
B.superior mesenteric
C.inferior mesenteric
D.rt renal vein
E.left renal vein

12.What is most appropiate about uterine tube

A.moves laterally to open medialy in the ovaries
B.present at the base of broad ligament
C.is 6cm long in adults
D.has an abd ostiium which is 0.5mm when dilated
E.extend above the fundus of uterus

13.Genital tubercle in females give rise to

A.labia majora
B.labia minora
C.clitoris
D.vestibular glands
E.bulbourethral glands

14.urachal cyst is the remnant of

A.allantois
B.mekel's diverticulum
C.........
D.....
E.........

15.Thymus develops from

A.ectoderm of 2nd brachial pouch
B.endoderm of 2nd brachial pouch
C.ectoderm of 3rd brachial pouch
D.endoderm of 3rd brachial pouch
E.mesoderm of 3rd brachial pouch

16.A pt presented with an opening in the ant wall of sternocledomastoid muscle with pussy discharge coming out of it.this problem occured due to abnormality in one of the following

A.1st pharyngeal cleft
B.2nd pharyngeal cleft
C.1st pharyngeal pouch
D.2nd pharyngeal pouch
E.3rd pharyngeal cleft

17.Diaphragmatic hernia occurs due to

A. absence of pleuro-pericardial membrane
B. absence of septum transversum
C. absence of pleuro-peritoneal membrane
D. absence of central tendon
E. absence of arch?

18.Erythropoises occurs in middle trimester mainly from

A.liver
B.kidney
C.spleen
D.bone marrow
E.both liver n kidney
19.Superior parathyroid gland develops from

A.1st brachial arch
B.2nd brachial arch
C.3rd brachial pouch
D.4th brachial pouch
E.3rd brachial cleft

20.In cervical carcinoma,wat is the lymph drainage

A.internal iliac
B.external iliac
C.both internal n external iliac
D.superficial inguinal
E.deep inguinal

21.RT coronary artery

A.arises from anterior aortic sinus
B.enters the rt heart border after passing through sulcus on posterir surface
C. supplies both ventricles
D. supplies S.A node
E. supplies left atrium

22.Fat deposition n ductal development of breast is the function of which hormone

A.estrogen
B.progesterone
C.prolactin
D.oxytocin
E.hcG

23.Inhibition of ovulation is the function of which hormone

A.progesterone
B .prolactin
C.dopamine
D.estrogen
E.GnRH

24.Thirst is least stimulated by

A.decrease blood volume
B.angiotension 2
C.salt depletion
D.decrease plasma volume
E.blood pressure

25.Most imp hormone involve in gluconeogenesis is

A.growth hormone
B.insulin
C.thyroxine
D.cortisol
E. Glycogen

26.A pt presented with rt iliac fossa pain. he is diagnosed to have acute appendicitis on flexing his rt thigh inward n medially he cries with pain.wat is the type of appendicitis he has

A.paracolic
B.retrocolic
C.pelvic
D.paracecal
E.subcecal

27.which of the following structure has both intra n extra pelvic extension

A.broad ligament
B.ovarian ligament
C.transverse cervical ligament
D.round ligament of ovary
E.round ligament of uterus

28.Stab wound above the piriformis muscle will damage which structure

A.superior gluteal nerves?
B.inferior gluteal nerves
C.sciatic nerve
D.tendon of obturator internus
E.inguinal nerves

29.Nerve supply of rectum

A.hypogastric plexus
B.greater splanchnic nerves
C.pelvis splanchnic nerves
D.short spanchnic nerves
E. inferior mesenteric plexus
30.One slide of a specimen shows stratified squamous epithelium with vascular smooth muscle and elastic fibres most probably the specimen is

A.cervix
B.uterine tube
C.vagina
D.uterus
E.anus

31.Apoptosis is inhibited by

A.capsases activation
B. Bcl-2 inhibition
C. Activation of P-53
D. apoptosis related protein ??
E…


32.Most potent chemotactic factor is

A.C3a
B.C5a ?
C.IL-1
D.viruses
E.arachdonic acid metabolites

33.what is the most appropiate answer

A.C3a is opsonin
B.C5a is potent anaphylactic
C.C5a is a potent chemotactic agent
D.C3b n C5a are anaphylactant
E.......

34.Low serum complement seen in which condition

A.SLE
B.ankylosing spondiolitis
C.rheumatid arthritis
D.multiple sclerosis
E.dermatomiositis

35.Pt presented with photosensitivity,rash n joint pain.which is the daignostic in this case

A.anti SS antibodies
B.anti DS antibodies
C.RA
D.anticentromere antibodies
E.antimitochondrial antibodies

36.Most dangerous Side effect of estrogen therapy is

A. suppression of lactation
B. hirsutism
C.thrombophlebitis
D.osteoporosis
E..........

37.If a pt is on long term corticosteroid therapy.she wil most probably hav increased risk of

A.repeated infections
B.decrease wound healing
C.osteoperosis n fractures
D.........
E....

38.Platelets r stored at room temprature because

A.to optimise its function?
B.to maintain its number
C.to avoid allergic reaction
D. to increase number
E.....

39.Which is most imp indicator of muscle protein loss

A.urea
B.ammonia
C.none of above
D.uric acid
E. ?

40.wat is true about DNA

A.Double stranded in which two strands are antiparallel to each other
B. a single turn of DNA is 2nm
C.Can exist as helix in single stranded form as well as in Double stranded form
D.
E.
41.A normal looking Girl came to you with primary amenorrhea,on examination she was having short blind vagina wid normal vulva, scanty pubic n axillary hairs,normal breasts,absent uterus,her karyotype is?

A.46XY
B.47XY
C.47XXY
D.45XO

45.A pt came to you with primary amenorrhoea,webbed neck,short stature diagnosed as Turner syndrome, wht is her karyotype?

A.45XO
B.46XY
C.47XXY
D.46XX
E.......

46.Hydrolysis of DNA gives

A.proline
B.ribose
C.phosphoric acid?
D...
E...

47.Which of the following has highest cholesterol content

A. Chylomicron remnants
B. LDL
C. VLDL
D. HDL
E. IDL

48.Which hormone convert to its active form at target tissue with the help of 5 a reductase?

A. Testosterone
B.Estrogen
C.dihydoepiandosterone
D.Progesterone
E...

49.Which drug do not cause gynaecomasia?

A.Digoxin
B.Griseofulvin
C.Cimetidine
D.Androgen
E.spironolactone

50.Which is the best option in identification of Turner's syndrome

A.ultrasound
B.barr bodies
C.karyotyping
D.Ct scan
E...

51.Pyruvic acid is used as intermedite in which step of cell metabolism

A.carbohydrate to acetyl co A
B.glucose to acetyl co A
C.acetic acid to acetyl co A
D.......
E..

52.Epinephrine,norepinephrine n dopamine are derivatives of which aminoacid

A.alanine
B.arginine
C.proline
D.tyrosine
E.phenylalanine

53.End product of purine metabolism is

A.urea and water
B.uric acid
C.glucose
D.water and ammonia
E.ammonia

54.Erythropoitin is secreted from

A.liver cell
B.peritubular interstitium of kidney
C.macula densa of kidney
D...
E...

55.ESR is decreased with

A.temprature
B.albumin is decreased
C.globulin is decreased
D.decreased blood volume
E.decreased plasma volume

56.RBC's are

A.biconvex
B.nonfragile
C.have nucleus
D.largest of all cells
E.have glycolytic enzyme activity
57.Tamoxifen has effect on which system

A.CVS
B.CNS
C.breast
D.liver
E.kidney

58.Which of the following drug is most notorious for causing renal toxicity

A.cisplatin
B.cyclophosphamide
C.vincristine
D.blephamide
E.....

59.Commonest Cause of PDA

A.drugs
B.toxin
C.permaturity
D.congenital abnormality
E..infection

60.Most common Cause of metaplasia is

A.ultrasonography
B.chronic irritation
C.infections
D.radiations
E...

61.Wat is the differentiating point of hyperplasia fron hypertrophy

A.increase in size of cell
B.increase in size n no of cell
C.is reversible
D.is premalignant ?
E.involve viscera only

62.Dysplasia is seen mainly seen in

A.bone marrow
B.cartilage
C.epithelia
D.fibrous tissue
E.deep tissue

63.Wat is the differentiating feature of hyperplasia from benign tumor

A.increase in no of cell
B.are the same as parental cell
C.cytplasm n nucleus ratio is derranged
D....
E...

64.What is the most true among following

A.mRNA has a codon
B.mRNA has anticodon
C.rRNA is most abundant n involve as ribosomes on endoplasmic reticulum
D.tRNA is largest
E.tRNA involve in protein synthesis

65.deficiency of THAIMINE(B1)causes all except

A.cardiac anomalies
B.muscle wasting
C.defects in alcohlics
D.subacute degenaration of spinal cord
E.cns abnormalities in alcoholics

66.AIDS is associated with all of the following except

A.kaposi sarcoma
B.lymphoid tumors
C.primary tumor of brain
D.STD
E.leukemia

67.VITAMIN involve in collagen synthesis is

A.ascorbic acid
B.biotin
C.thiamine
D.pyridoxine
E.niacin

68.Neural tube defects occur due to deficiency of

A.iron
B.folic acid
C......

69.Iron is stored in the form of

A.ferritin
B.transferrin
C.apoferritin

70.CO2 is tranported in lungs through
A.simple diffusion
B.fasilitated diffusion
C.active transport
D.carriar proteins
E.channels

71.HSV is associated with
A.CA cervix
B.CA vulva
C.vaginal wart
D.vulval papules?

72.If a pt has presented with the wart on lateral wall of introitus.the most probable daignosis will be
A.fibroepithelial lesion
B.squamous cell CA of skin
C.polyp
D.squamous CA of vagina
E.HPV

73.Pt presented on 18th day of menstrual cycle her normal cycle is such dat she ovulates on day 14. Which phase will she be in?
A.follicular
B.ovulatory
C.pre mestural
D.proliferative
E.secretory?

74.Optic groove appears on left side of forebrain on day
A.12
B.22
C.32
D.42
E.52

75.Germen measles causes which abnormality. What is the most appropriate answer
A.congenital cataract
B.congenital deafness?
C,cardiac anomalies?
D.limb deformaties
E........

76.A newborn with anterior abd wall swelling and umbilical cord attached to it.wat is the condition called
A.omphalocele
B.gastroschiaosis
C.umblical hernia
D.omentocele
E.intestinal malformation

77.PT has exophthalmos n his T3 n T4 are increased.wat is the option for treatment of exophthalmos
A.drugs blocking the action of T4?
B.direct suppresion of T4
C.hypophysectomy
D.administration of testosterone
E.partial parathyroidectomy

78.Treatment of diabetes in pregnant lady is
A.sulphonyl urea
B.insulin
C.biguanides
D.suphonylurea n biguanides
E...

79.Statistical average applicable to the measurement of a population is
A.mean?
B.mode
C.median
d.standard deviation
e.central

80.Two groups one with CHD and associated risk factors is compared to another group with CHD but no associated risk factors. What type of study is it?
A. case control
B. cohort
C. prospective
D. retrospective
E. random

81.Oxytocin is given to a pt in labour.it has effect on:
A. oxytocin receptors on cervix
B.oxytocin receptor on myometrium and endometrium
C.dec prostaglandin recp on cervix
D.dec prostaglandin recp on myometriun


82.Beta lactum acts through
A.cell membrane
B.cell wall
C.DNA
D.30S
E.50S

83.side effect of streptomycin
A.impairment of hearing
B.
C.
D.
E.

84.Which of the following is opportunistic organism
A.E coli?
B.salmonella
C.shigella
D.klebsella
E.chlostridium

85.Psudomembranous colitis is caused by
A.Cl.difficile
B.Cl.tetani
C.CL perfiranges
D.Cl.botulinum
E..

86.About aspargillosis which is not correct
A.is caused by fungal
B.usually seen in preexisting lung diseases?
C.is caused by long term antibiotic use.
D...
E........

87.Most common cause of puerperal sepsis is
A.bacteriodes
B,E coli
C.trichomonas
D.chalymadia
E.gonorrhea

88.Another question associted with aspargilosis

89.Inutero infection is not caused by
A.syphlis
B.mumps
C.cytomegalovirus
D.HIV
E.rhinovirus

90.A pt with cervical lymphadenopathy. he has granuloma on histopathology.wat is the diagnostic finding in this case for T.B
A.AFB
B.caseous necrosis
C.montoux test
D.x-ray
E.......

91.A 25 yrs old multiparaous presented with lower abd pain.on histology it showed solid n cystic areas intermixed but on laproscopy it was semi solid with brown centre.wat is the most probable daignosis
A.dermoid cyst
B.endometrial cyst
C.ovarian CA
D.enodmetrial CA
E.teratoma

92.A pt presented with primary amennorhea.well developed secondary sexual characterstics,present breast but absent uterus.the organ not found is due to abnormal development of
A.mesonephric duct
B.paramesonephric duct
C.genital tubercle
D.........

93.A woman presented with vulvar itching n pinkish purulent discharge.wat is the cause
A.trichomanas vaginilis
B.chlamydia
C.gonorhea
D.lactobacillis
E....

94.presense of delta antigen shows( HDV)
A.Infectivity
B.recovery
C.Chronicity
D. Acute phase
E.Measure of severity/fatality of the disease

95.Most common organism involve in post operative abdominal & gynaecological infection is
A.E coli
B.bacteroides
C.chlamydia
D.trichomonas
E.gomorrhea

96.A pt presented with lower abd pain n purulent white dischrge.om laproscopy fallopian tubes were sticked together n ruptured during operation.wat is the condition involved
A.acute salpingitis
B.chronic salpingitis
C.pyosalpinx
D.hydrosalpinx

97.Pt has a history of infection 4 weeks back.now he developed same pain again.wat type of cell will present at this time of disease
A.neutophils
B.eosinophils
C.lymphocytes
D.basophils
E.mast cells

98.What is the drug of choice for DVT in ist trimester of pregnancy
A.aspirin
B.heparin
C.warfarin
D.indomethacin
E.mefanamic acid

100.Deficiency of Vit K will effect all of the following except
A.factor 7
B.factor 5
C.factor 9
D.fibrinogen
E.prothrombin

101.At which step intrinsic n extrinsic pathways converge
A.activater 10
B.activated 11
C.activated 12
D.prothrombin
E.activeted 9

102.Type 2 hypersensitivity reaction includes
A.erythroblastis fetalis
all other options were wrong

103.Transfusion reaction will not occur in a pt if we transfuse the
A.group A with O
B.group A with AB
C.group AB with A
D.group B with O
E.group O with B

104.Which of the following does not cross placenta easily
A.glucose
B.amino acid
C.Ig G
D.Ig M
E.CO2

105.Which drug is used in the treatment of hirsutism
A.antiandrogens
B.estrogen
C.testosterone
D.cryptoterone acetate
E.diacrone

106.Most common organism involved in infection caused by IUCD
A.actinomycosis
B.E coli
C.bacteroide
D.gonorrhea
E.trichomonas

107.ECF differ from ICF in
A.high conc. of Na n Ca
B.high K low Na
C.low Na n Ca high K
D. high conc. Of K & Ca with low Na
E.Volume …

108.diffrence of smooth n skeletal muscle
A.Ca-Calmodulin complex is present in smooth muscles

109.In partial mole,karyotype is
A.46XX
B.45XO
C.47XX
D.47XY
E.69XXY

110.If carbohydrate is separated from proteins wat will be the fate
A.aggregation n precipitation
B.eliminate fron kidney?
C.increase uptake int muscle

111.Lysozome contains
A.glucose
B.Ca
C.Na
D. hydrolases
E.alkaline phosphate

112.Which of the following does not have sulpha group
A.cysteine
B.cystedine
C.homocystine
D.methionine
E.threonine

113.What is not true about levator anni muscle
A.supplied by L1 n L2
B.puborectalis forms part of it

114.What is not true about ureter
A.enter the pelvis infront of common iliac artery
B.enter the bladder on superio lateral surface of trigone
C.not hav sphincteric valve at opening into bladder
D.can be palpated at pelvic brim
E.??
115. MCQ on Significance
A.

116. If a test is able to both diagnose and eliminate diseased and non-diseased wht is it called?
A.Specific
B.Accurate
C.Sensitive
D…

117. From a population two groups are selected on age basis and then subgroups are assigned randomly for the study.wht type of sampling is it?
A.Multistage sampling
B.Stratified sampling
C.Random stratified sampling
D.simple random sampling

118. Metabolic Alkalosis is caused by
a) Chronic Hypoventilation
b) Pancreatic Fistula
c) hyperglycemia
d) ATN

119.Q on Metabolic acidosis

120.Q on compensated respiratory alkalosis

121. For Rh blood transfusion to the anemic baby
A.crossmatch blood to babys plasma
B.Crossmatch blood to mothers plama
C.Crossmath blood to mothers blood
D.give wbc depleted transfusion
E.Give red cell concentrates crossmatched wid fetal blood

122. Following is Autosomal Dominant Disorder
a) Phenylketonuria
b) Hereditary Spherocytosis
c) Hemophilia A
d) Cystic Fibrosa

123. Source of Estrogen & Progesterone in last trimester is
a) Chorionic Villus
b) Placenta
c) Overy
d) Adenohypophysis
e) Corpus Leuteum

124. About K - Regulation , following is correct
a) K is 98% in the cells
b) normally completely reabsorbed in Distal tubule
c) increase water affects K-balance?
d) decrease in D.K.A

125.On examination of a female,a plaque like lesion seen on Postero-superior aspect of Vagina,it is
a) Squamous Cell CA
b) Adenocarcinoma
c) Rhabdomyosarcoma
d) wart?



Gyn Obs July 07
GYN OBS Questions JULY 2007

Hi just got to know of dis site through cpsp community otherwise would have mailed u questions way back anyways great effore am mailing fcps part 1 obs gynae mar n jul 2007

question no 1:eversion of endocervical epithelium in pregnancy is due to:
a)physiological change
b)chronic cervicitis
c)carcinoma in situ
d)estrogen stimulation
e)epidermadization of epithelium


2:primary oocyte surrounded by a non cellular layer:
a)theca interna
b)theca externa
c)corona radiata
d)cumulus oophurus
e)zona pellucida

question no 3:
simple columnar epithelium is the lining of:
a)epithelium lining the ducts
b)cornea
c)testis
d)uterine tubese)

question no 4:48 years lady has carcinoma cervix that has involved the cervix and has involved the vagina but not its lower third and has not extended onto pelvic walls stage of ca:
a)stage 1ab)stage 1bc)stage 11d)stage 111e)stage 4
question no

5:38 years lady has endometrial ca that has involved upto half the tickness of myometrium and ovaries are normal stage:
a)stage 1a
b)stage 1b
c)stage 1c
d)stage 11
e)stage 111

question no 6:20 yrs old girl has been raped,she has now watery vaginal discharge,has painful red papules n ulcers on vagina and vulva has fever and tender inguinal lymph nodes,causative organism:
a)bacteroides
b)chlamydia
c)herpes simplex
d)gonococcus
e)trichomonas vaginalis

7: a married woman has right sided tubulo ovarian abscess which was removed in a surgery had an iucd placed before,causative organism:
a)chlamydia
b)gardenella
c)tuberculous
d)bacteroides
e)gonococcus

8.the late secretory phase of menstrual cycle is not characterised by:
a)the spongy layer becomes thick
b)corpus luteum secretes estrogen n progesterone
c)the endometrial thickness is doubled
d)endometrial cells have subnuclear vacuolations
e)endometrial glands become tortuous

9.just 2 days before ovulation,LH is increased:
a)2 fold
b)2 to 4 fold
c)2 to 6 fold
d)6 to 8 fold
e)8 to 10 fold

10.menopause is characterised by:
a)atrophy of uterus
b)decrease in ovarian size
c)derease estrogen increased FSH AND LH
d)decreased estrogen increased FSH decreased LH

11.sperms stay active in female genital tract:
a)12 hours
b)12 - 24 hours
c)12 -36 hours
d)24 - 48 hours
e)24 - 72 hours

12.the volume of distribution of a drug is not influenced by:
a)age
b)sex
c)cardiac failure
d)renal failure

13.the bioavailability of a drug determines :
a)efficacy
b)efficency

14.paracetamol:
a)is more potent analgesic than codeine
b)is a weak anti inflammatory agent
c)overdosage can cause nephrotoxicity

question no 15:anticoagulants are contraindicated in:
a)cerebral infarction
b)thrombocytopenia

16.20 yrs lady has endotoxic shock after septic abortion,which of the following investigation is least likely to benefit:
a)clotting time
b)FDPs
c)prothrombin time
d)platelet counte)serial fibrinogen levels

17.30 yrs male has dyspnea on lying down,examination revealed:
a)multinodular goitre
b)retrosternal goitre
c)colloid goitre
d)cold nodule

18:cortisol:
a)increases amino acids n glucose in blood
b)is a glucocorticoid
c)is found in circulation bound to proteins
d)metabolised in liver
e)formed from cholesterol

19.JVP is differentiated from carotid pulse:
a)does not change with abdominal pressure
b)does not change with posture
c)does not change with respiration
d)pulsation varies with respiratione)prominent outward movement

20.a multigravida admitted with labour pains examination revealed os 6 cm dilated,baby cephalic,and head at zero station,the point of reference is:
a)ischial tuberosity
b)pubic arch
c)pubic symphysis
d)ischial spines
e)sacral promontory

21.at LSCS the artery damaged in superficial fat in suprapubic incision:
a)inferior epigaSTRIC artery
b)superior epigastric artery
c)internal iliac artery

22.following is not a tumor marker:
a)acid phosphatase
b)beta hCG
c)alpha fetoproteins
d)CEA
e)placental alkaline phosphatase

23.question o 23:beta hCG:a)it is a polypeptideb)it has action similar to FSHc)it prevents involution of corpus luteumd)e)

24:adrenal medulla will selectively secrete norepinephrine in the condition:a)exerciseb)sleepc)cold exposured)hemorrhagee)

25:cardiac output is decreased in:a)pregnancyb)standing or sitting from lying positionc)exercised)e)

26:cardiac output is decreased in all except:a)sleepb)starvationc)sittingd)e)

27:epinephrine will selectively onstrict muscles in:a)arteriolesb)bronchiolesc)ciliary muscled)pupilse)intestinal walls


28:spores should be disinfected by:a)dry heat at 100cb)dry heat at 60cc)dry heat at 160cd)cidex solutione)moist heat

29:following is not a disinfectant:
a)derivatives of salicylic acid
b)alcohol
c)soap
d)
e)

question no 30: that she had urinary incontinence,due to injury and communication in:
a)ureter n bladder
b)ureter n vagina
c)bladder n cervix
d)bladder n vagina
e)bladder n uterus

question no 31:perineal body is injured,muscle damaged is:
a)bulbospongiosis n superficial perineal muscles
b)superficial n deep perineal muscles
c)external anal sphincter n deep perineal muscles
d)ischiocavernosus n sphincter urethrae
e)sphincter urethrae n deep perineal muscles

32:pudendal nerve:
a)formed from ant primary rami of second third n fourth sacral spinal nerves
b)crosses the ischial spine medial to internal pudendal artery
c)enters the pudendal canal on the lateral side of ischiorectal fossa
d)leaves the pelvis by passing through the upper part of greater sciatic foramen
e)reenters the pelvis through lesser sciatic foramen

33:following does not form wall of inguinal canal:
a)aponeurosis of external oblique
b)internal obliquec)conjoint tendon
d)lacunar ligament
e)fascia transversalis

34:following does not pass through lesser sciatic foramen:
a)inferior gluteal vessels
b)tendon of obturator internus
c)nerve to obturator internus
d)pudendal nervee)internal pudendal artery

35:after LSCS patient had peritoneal hemorrhage she was reopened bleeding most likely from:
a)branch of ovarian artery
b)branch of uterine artery
c)internal iliac artery
d)obturator artery
e)external iliac artery

no 36:left ovarian artery is a branch of:
a)abdominal aorta
b)left renal artery
c)uterine artery
d)left inferior mesenteric arterye)obturator artery

question no 37:patient had laparotomy for ovarian mass mass was burried in ovarian fossa artery likely to be injured is:a)common iliac vesselsb)internal iliac vesselc)external iliac vesseld)ovarian arterye)obturator artery

questoion no 38:insulin stimulates transport of glucose into:
a)renal tubular cells
b)neurons of cerebral cortex
c)skeletal muscled)intestinal cells
e)liver

question no 39:the secretion of following is not increased in pregnancy:
a)cortisol
b)growth hormone
c)thyroid hormone
d)prolactine)PTH

question no 40:a patient had hypophsectomy for pituitary tumor she has amenorrhea for 8 months ovulation induction can be done in her by:
a)clomiphene citrate
b)pulsatile hCG
c)hMG
d)hMG followed by pulsatile hCG

question no 41:following does not occur in pregnancy:
a)increase in fasting blood sugar
b)increase in uric acid level
c)increase in serum urea level
d)decrease in serum albumine)decrease in serum proteinse)decrease in nitrogen levels

question no 42:respiratory acidosis occurs in:
a)emphysema
b)hyperventilation
c)ingestion of excess amount of sodium bicarbonate
d)starvatione) vomiting

question no 43:CO2 is transported from the alveoli of lungs into blood in lungs via:
a)diffusion
b)active transport
c)facilitated diffusion
d)secondary active transporte)osmosis

question no 44:O2-Hb dissociation curve delivering O2 in normal limits will have:
a)PaO2 40mmHg SO2 60%
b)PaO2 68mmHg SO2 78%
c)PaO2 96mmHg SO2 90%
d)PaO2 123mmHg SO2 98%
e)PaO2 256mmHg SO2 99%

question no 45:b-hCG has alpha n beta units,each unit has aminoacids:a)102 alpha 210 betab)100alpha n 160 betac)92 alpha n 156 betad)e)

qyuestion no 46:mothers milk differs from cows milk in that it contains:a)calciumb)lactosec)proteinsd)vitamin D

question no 47:pelvic outlet:
a)can b increased in supine position
b)begins at the level of ischial spines
c)dimensions change by movements of coocyx
d)contracture is more common than mid cavity probleme)

question no 48:patient has endocrine disorder has metabolic alkalosis and hypokalemia,he has excess of:a)ACTHb)aldosteronec)cortisold)estrogene)progesterone

question no 49:hormone that stores carbohydrates, lipids n proteins in cells is also known as hormone of abundance:a)growth hormoneb)cortisolc)thyroid hormoned)estrogene)progesterone

question no 50:hyperglycemia is caused by:a)insulinb)glucagonc)estrogend)progesteronee)growth hormone

question no 51:ketoacidosis is caused by:
a)increased production of glucose
b)decreased production of glucpoose
c)decreased utilisation of carbohydrate in body
d)insulin excess

question no 52:insulin secretion is stopped by:a)glucagonb)decreased k levelsc)somatostatind)e)

question no 53:hormone that stores carbohydrates,lipids n proteins in body is:a)growth hormoneb)thyroid hormonec)cortisold)insuline)PTH

question no 54:8 yrold girl has vaginal bleeding she is taller than her age and examination revealed an abdominal mas arising from pelvis she is suffering from:
a)constitutional; symptoms
b)hypothalamic tumors
c)hypothalamic mass
d)granulosa cell tumor
e)adrenal gland tumor

question no 55:least positive titre for widal test isa)1:80b)1:160c)1:260d)1:320e)1:156


question no 56:all of the following can b stored at 4c except:a)shigellab)vibrio cholerac)d)e)

question no 57:vaccine is available for all except:a)EBVb)typhusc)pneumococcusd)e)

question no 58:patient has presented with fever n generalised lymphadenopathy, bone marrow exam normal,weight loss positive,he is suffering from:a)metastatic carcinomab)hogkins lymphomac)non hogkins lymphomad)CMLe)

question no 59:patien is taking glucocorticoids for 10 days,she wont suffer from:a)moon faceb)hypotensionc)thin skind)e)

question no 60:gestational age can best be estimated from:a)abdominal circumference at 36 wksb)biparietal dia at 28 wksc)crown rump length at 8 wksd)femur length at 32 wkse)

question no 61:turners syndrome,most definitive diagnostic test:a)presence of barr bodyb)chromosomal analysis

question no 62:14 yrs girl has primary amenorrhea,normal breast development,secondary sexual characteristics normal,short blind ending vagina,slight pubic hair,usg revealed absent uterus,underlying cause:
a)congenital adrenal hyperplasia
b)chromosomal defect
c)failure f development of genital tubercle
d)defect of paramesonephric duct

question no 63:the lumen of uterine cavity is obliterated by fusion of:
a)decidua basalis n spongy layer
b)decidua basalis n decidua capsularis
c)decidua capsularis n decidua parietalis
d)decidua parietalis n decidua basalise)

question no 64:a woman has been exposed to radiation in young life,now she has visited a gynaecologist after some years,organ most likely to b affected:a)fallopian tubesb)cervixc)ovaryd)endometriume)vagina

question no 65:repeated blood transfusions (AB n Rh compatible) in an individual will lead to:a)haemochromatosis of liverb)hypocalcemiac)haemoglobinuriad)transfusion reactione)

question no 66: Rh incompatibility occurs in a mother if she has:a)Rh negative fetusb)Rh positive fetusc)d)e)

question no 67:the best way to prevent Rh isoimmunisation in a woman who has given birth to Rh positive fetus:a)blood transfusionb)platelet transfusionc)serumd)Rh immunoglobulinse)

question no 68:a person brought in er had burns,he was admitted later on he developed fever and bleeding diathesis,he is suffering from:a)DICb)hepatorenal failurec)hepatic failure of prothrombin productiond)renal failuree)

question no 69:following is not a cause of DIC:a)pre eclamsiab)amniotic fluid embolismc)cervical carcinomad)abruptio placentaee)PPH

question no 70:following is not a cause of DIC:a)abruptio placentaeb)burnsc)malignant tumorsd)e)

question no 71:multigravida P9+0 delivered a boy in a village 2 hours later she started bleeding profusely had oozing from gums she was brought to hospital her labs showed hb 6 TLC 15000/ul platelets 30000/ul she is suffering from:a)DICb)PPHc)clotting disorderd)bleeding diathesise)ITP

question no 72:27 year old female had profuse vaginal bleeding 2 hrs after delivery her labs showed hb 6 TLC 15000/ul plt 30000/ul peripheral blood film showed burr bodies,she is suffering from:
a)DIC
b)PPH
c)ITP
d)von willebrand disease

question no 73:vit k directly influences d reaction:a)conversion of fibrinogen to fibrinb)prothrombin to thrombinc)activation of factor 7d)formation of fibrinogen

question no 74:intrinsic n extrinsic pathways converge upon:a)activation of factor 7ab)activation of factor 5ac)factor 12ad)prothrombine)tissue factor

question no 75:bacteria that grow at 42c:a)vibrio cholerab)gonococcusc)shigellad)Ecolie)salmonella

question no 76:following is not true about endocarditis:a)blood cultures for causative organisms r usually negativeb)is caused by streptococcus group of bacteriac)is more common in elderly groupd)valvular involvemente)

question no 77:cause f subacute bacterial endocarditis:a)H.influenzaeb)Ecolic)streptococcus viridansd)e)

question no 78:interferon:a)binds to DNAbbinds to RNAc)is a protein derivatived)inhibits protein synthesis n their phosphorylation

question no 79:highest energy compound:a)ATPb)glucosec)hexose phosphated)starche)glycogen

question no 80:the end product of carbohydrate metabolism:a)fructoseb)glucosec)galactosed)starche)pyruvate

question no 81:L-dopa taken by a patient will decrease :a)estrogenb)progesteronec)prolactind)cortisole)

question no 82:isoniazid:a)cant b administered with rifampicinb)is contraindicated in nephropathyc)pyridoxine is used prophylactically to prevent its neurotoxic effectsd)e)

question no 83:hepatitis b following is not true:a)has an incubation period less than that of hep ab)caused by sexual contactc)caused by DNA virusd)e)

question no 85:actinomyces does not cause disease in:a)boneb)lungc)brain

question no 86:disseminated coccidiodomyces following is not true:a)infection will not provide life time immunityb)c)d)e)

question no 87:middle aged man crossing the road stumbled n hit by a car he fell down had injury to neck of fibula of his left leg was taken to hosp examination revealed that he is unable to dorsiflex and evert his left foot the injury is to:
a)common peronel nerve
b)tibialis post
c)superficial peroneal nerve
d)deep peroneal nervee)

question no 88:ECF:a)provides nutrients to cellsb)provides glucose to cellsc)bathes the cellsd)takes away electrolytese)takes away wastes

question no 89:spermatogenesis is stimulated by:a)FSHb)testosteronec)LHd)FSH n testosteronee)

question no 90:patient brought in shock in er successful fluid replacement will b indicated by:a)decrease in tachcardiab)increase in bpc)increase in urine outputd)e)

question no 91:cardoiac cycle:a)60% of ventricular filling occurs in diastoleb)end diastolic left ventricular vol is 100 mlc)ejection fraction is 75%d)the notch of d pulse is due to atrial systolee)

question no 92:a man has a pulse rate of 72/min impulse originates in:a)AV nodeb)SA nodec)left bundle branchd)purkinje fibrese)

question no 93:muscles r derived from:a)mesodermb)ectodermc)endoderm

question no 94:in our country d cause of cell inujury is:a)alcoholb)hypoxiac)ischemiad)ionising radiatione)malnutrition

question no 95:in our country cause of fatty liver is:a)alcoholismb)amoebiasisc)due to hep b n cd)increase intake of fatsdecreased protein utilisation

question no 97:in d female d neck of urinary bladder is located :a)in front of rectumb)below d urogenital diaphramc)in d urogenital diaphramd)above d urogenital diaphrame)behind symphsis pubis

question no 98:the highest content of triglycerides is in:a)chylomicron remnantsb)HDLc)LDLd)
IDLe)VLDL

question no 99:following is not a phospholipid:a)cephalinb)lysolecithinc)lecithind)plasminogene)sphingolipid

question no 100:a human being can survive without:a)calciumb)proteinsc)carbohydratesd)lipidse)

question no 101:patient has pyuria but has negative urine culture for organisms this happens in all conditions except:
a)acute post streptococcal glomerulonephritis
b)polycystic kidneys
c)urine stored at room temp for long
d)alkaline urinee)vesical calculus

question no 102:broad ligament a double layered fold of peritoneum ,ovary is attached to it by:
a)mesoovarium
b)suspensory ligament of ovary
c)round ligament of ovary
d)cardinal ligament



Gynae Obs Paper July 2007 Part-1

1. Eversion of cervix during pregnancy occur in response to:
a) Estrogen
b) Physiological change
c) Ca Cervix
d) Gonadotropins
e) Cervicitis

2. If Carcinoma involves nipple of breast which lymph nodes r likely
to involve first?
a) Pectoral
b) Apical
c) Central
d) inernal mammary

3. A Pt. came in OPD With enlarged Ant.Axillary lymph nodes and diagonosed a case of Ca Breast , which region is most likely involved?
a) Upper Outer Quadrant
b) Lower Inner Quadrant

4. More or less same question as # 3. reapeated again

5. A pt came to you with primary amenorrhoea,webbed neck,short stature diagnosed as Turner syndrome, wht is her karyotype?
a) 45XO
b) 46XY
c) 47XXY

6. A normal looking Girl came to you with primary amenorrhea,on examination she was having short blind vagina wid normal vulva, scanty pubic n axillary hairs,normal breasts,absent uterus,her karyotype is?
a) 46XY
b) 47XY
c) 47XXY

7. Wht is the best way to detect exact gestational age of fetus?
a) CRL at 8 wk.
b) Femur length at 28th wk.
c) Biparietal diameter at 18th wk.
d) Abdominal circumference at.....

8.Sperms life span in female genital tract is ?
a) 24 to 48 hrs
b) 24 to 72 hrs

9. Spermatogenesis is controlled by?
a) Testosterone
b) FSH+ Testosterone
c) LH
d) Estrogen

10. Almost same as above

11. In drug receptor relations which one is incorrect?
a) Bradycardia --- Propranolol
b) Miosis --- Pilocarpine
c) Skeletal Muscle Paralysis --- Tubocurarine
d) Bronchodilation --- Salbutamol

12. Which one of them do not pass through Lesser Sciatic Foramen?
a)Tendon of obturator internus.
b)Pudendal Nerve
c)Internal Pudendal Vessels
d)sup. & inf. gluteal vessels

13. In a motorbike accident neck of fibula fractured and lead to a loss of dorsiflexion n eversion of foot,which nerve is likely to b damaged?
a)Common Peroneal
b)Deep peroneal
c)Superficial Peroneal

14. Wht is most likely to b true about pudendal nerve?
a) Enter the ischiorectal fossa thru its lateral wall
b) Leave the pelvis thru upper part of greater schiatic notch
c) Re enter the pelvis thru lesser schiatic notch

15. If perineal body is cut which muscle likely to b damaged?
a) Bulbospongiosus + superficial Transverse Perieal
b) Deep + Superficial Transverse Perineal

16.Ureter is found at :
a) the bifurcation of common iliac artery
b) pelvic brim

17. If ovaries are removed frm ovarian fossa, wht is most likely to b damaged?
a) Internal iliac
b) uterine artery
c)Ovarian artery
d) External iliac artery

18. To reach the uterus, uterine artery crosses the:
a)Ureter
b)Fallopian Tube
c)Ligament of ovary


19. If Ca.Cervix metastasize to Labia majora which one of these is most likely responsible for spread?
a) Round lig. Of uterus
b) pubocervical ligament
c) pectineal ligament

20)which structure do not participate in forming the walls of inguinal canal?
a) Transversalis fascia
b) Internal Oblique
c) External Oblique
d) Conjoint tendon
e) Lacunar ligament

21) If after 1 hr of ceaserian section pt start b bleeding internally which one is most likely damaged?
a) Uterine artery
b) Internal iliac a.
c) Ovarian artery
d) uterine vein

22) If on lying down pt became dyspnoec wht is the cause?
a) Retrosternal goiter
b) Multinodular goiter
c)

23) If a women at 10th week pregnancy hav Hb 12;Hct 25 Normocytic Normochromic film but at 32 wk pregnancy her Hb is 10.9 ; Hct 20 with same peripheral film wht is the cause?
a) Physiological change
b) Iron deficiency
c) Folate Deficiency
d) Vit B12 deficiency

24) Ovaries are attached to broad ligament by?
a) Mesoovarium
b) Mesosalpinx
c) suspensory ligament

25) In females Neck of bladder ends :
a) above the urogenital diaphragm
b) below the urogenital diaphragm
c)

26) Most common cause of hosp. acquired infection is?
a) Indwelling catheters
b) I/V lines
c)

27) Which method is used 2 kill spores of surgical instruments?
a)dry heat at 160c for 1 hr
b)dry heat at 160 c for 4 hr
c)moist heat at 160c for1hr
d)Gension violet for 4 hrs

28) Which one is not used as insecticidal?
a) Derivatives of salicylic acid
b) Phenol
c) Sodium hypochlorite
d) Soap

29) Most common cause of subacute bacterial endocarditis is:
a) sterpt viridans
b) sterpt epidermidis
c) staph aureus

30) Food poisoning cause by staph. Is mainly due to:
a) exotoxin
b) endotoxin
c) enterotoxin

31) Now a days the best way to diagnose viral infection is:
a) ELISA
b) by detecting virus from secretion or wound
c) immunofloresence

32) If pt hav persistent pyuria all can be the cause except:
a) polycystic kidney disease
b) long standing alkaline urine at room temperature
c) Post Streptoccocus glomerulonephritis

33) wht is the least positive value for widal to diagnose typhoid?
a) 1:80
b) 1:120
c)1:360

34) Two days before ovulation LH surge rises to
a) 1 to 5 fold
b) 4 to 6 fold
c) 6 to 10 fold

35) villous surrounded by cyto n syncitiotrophoblast is:
a) Primary villous
b) secondary villous
c) tertiary villous

36)Following is not a proven cause of hirsuitism?
a) Androgen
b) PCOs
c) steven leventhal sundrome
d)

37) CO2 is transported frm blood to alveoli by :
a) Facilitated diffusion
b) Active transport
c) Diffusion

38) Active transport differs frm Passive due to:
a) depend on osmosis
b) require carriers but no energy
c) requires energy but no carriers
d) require energy by phosphate anhydride bond

39) Mean Arterial pressure is
a) difference b/w systolic and diastolic pressure
b) Diastolic + 1/3 pulse pressure
c)

40) Columnar epithelium is found in:
a) uterine tubes
b) Thymus
c) ovaries

41) Obese are at risk of all except:
a) Hyepertension
b) Diabetes
c) Thyrotoxicosis
d) IHD

42) Following will lead to Respiratory Acidosis?
a) emphysema?
b) heavy intake of NaHCO3
c) Vomiting

43) Hormone which causes an increase carbohydrate protein and lipid stores is?
a) GH
b) insulin
c) cortisol
d) Aldosterone
e) thyroxine

44) same question

45) insulin secretion is inhibited by:
a) glucagon
b) secretin
c)

46) Acellular layer surrounding an ovum is :
a) Zona pellucida
b)Granulosa layer
c) Theca interna
d) Corona radiate

47) Primordial germ cells arises frm:
a) Yolk sac

48) Highest energy containing compound is:
a) Starch
b) Glucose
c)Hexose phosphate
d) ATP

49) Which contain high triglyceride?
a) Chylomicron remnants
b) LDL
c) VLDL
d) HDL
e) IDL

50) Which is not a phospholipid?
a) Lecithin
b) ISolecithin
c) Cephalin
d) Plasmalogen

51)Low PO2 Stimulate respiratory centre thru:
a) Directly acting on medulla
b) stimulating peripheral chemoreceptor
c) acting on central chemoreceptor

52) If Rh -ve mother deliver an O +ve fetus wht is the best way to protect subsequent pregnancies?
a) Immunoglobulin

53) wht is the landmark for biscop scoring:
a) Ishial Spine
b) Ishial Tuberosity

54) In a women of craniopharyngioma wht is the best opetion to induce ovulation?
a) Clomiphen citrate
b) Menopausal Gonadotropins

55) If you give L Dopa to the pt of Parkinsonism which hormone will decrease?
a) Prolactin
b) FSH,LH
c) Testosterone

56) Which hormone convert to its active form at target tissue with the help of 5 α reductase?
a) Testosterone
b) Estrogen

57) Pt came to you with eosinopenia,polycythemia,lymphocytopenia and raised ACTH level,wht is most likely?
a) Cushing disease
b) Cushing Syndrome
c) Addisons disease

58)Wht change will not seen in a women having inc. cortsiol level?
a) Hypotension
b) Moon face
c) Buffalo Torso
d) Thin skin
e) Hirsuitism

59) Main support of uterus is:
a) Cardinal ligament

60) Rational for storage of platelet at room temp. is :
a) Optimize the platelet function
b) platelet number do not dec.

61) Women brought to you in ER in shock who had a 10 pint transfusion 10 yrs back due to PPH is most likely having:
a) Sheehan syndrome

62) Therapeutic index indicate:
a) Bioavailability
b) Efficacy
c)

63) If after 15 days of c-section due to Obstructed Labour pt. developed Urinary incontinence,wht is the most likely cause?
a) Vesicovaginal Fistula
b) Vesicouterine Fistula

64) Which one is the supporting ligament of uterus tht is palpable on Per-rectal examination?
a) Uterosacral ligament

65) If 35 yr old women developed Generalized Lymhadenopathy wht is the most likely cause:
a) Hodgkin Lymphoma
b) Nonhodgkin lymphoma
c) Malignant disease
d) CML
e) ALL

66) If women accidently exposed to radiation 3 to 4 yrs back which organ is most likely effected?
a) Cervix
b) Fallopian tube
c) Ovaries
d) Vagina

67) Wht is the best method to check bone density?
a) CT scan spine
b) MRI spine
c) Calcium Level
d) D scan of spine

68) Which drug do not cause gynaecomasia?
a) Digoxin
b) Griseofulvin
c) Cimetidine
d) Androgen
e) Estrogen

69) Pt developed Morning stiffness,Photosensitivity,rash which test will be positive?
a) RA
b) ANA
b) antimitochondrial

70) Wht is the end product of glucose breakdown ?
a) Galactose
b) Lactose
c) Fructose
d) Pyruvate

71) In a pre eclamptic pt which drug is used to lower BP before surgery?
a) Hydralazine
b) Amlodipine

72) Amniotic fluid embolism is related to:
a) DIC

73) Which hormone is necessary for brain development of fetus?
a) GH
b) Thyroid
c) Cortisol

74) If α feto protein is high wht is most likely?
a) Anencephaly
b) Spina bifida Occulta
c) Down syndrome

75) T3 , T4 raised in pregnancy due to:
a) Inc TBG
b) Inc TSH
c) Inc TRH

76) Which one is not a tumor marker?
a) HCG
b) CEA
c) α fetoprotein
d) Placental Acid phosphatase(PLAP)
e) Alkaline Phosphatase

77) Hysterosalpingography is more accurate than MRI in:
a) Adenomyosis
b) blocked tubes

78) TORCH can lead to all EXCEPT:
a) Pnuemonia
b) Cataract
c) Deafness
d) Cardiac anomalies
e) Blindness

79) DNA is:
a) Double stranded in which two strands are antiparallel to each other
b) all other wrong

80) Which antithyroid drug is given during Pregnancy?
a) Propylthiouracil

81) Tx of raised sugar in pregnancy is :
a) Insulin
b) Sulfonylurea

82) Menopause is diagnosed by:
a)Raised FSH,Raised LH,dec Estrogen
b)dec FSH n LH

83) Tuboovarian abcess caused by IUCDs…wht is the most likely causative agent?
a) Actinomycosis
b) Streptococcus

84) Which component is low in Cow`s milk as compared to mother`s milk?
a) Lactose
b) Fat
c) Calcium
d) Protein

85) Excess of Aldosterone will cause:
a) Hypokalemia
b) Hyperkalemia

86) Which method is most useful in identifying chromosomal abnormalities?
a) Karyotyping
b) U/S
c) Bar bodies

87) DIC is related to all EXCEPT:
a) Cancer
b) Amniotic fluid embolism
c) Sepsis
d) RPOCs
e) answer

88) In C-section insicion is given in subcutaneous fat suprapubically will likely to damage?
a) Superior mesenteric artery
b) Deep Circumflex iliac artery
c) Superior epigastric artery
d) inferior epigastric artery
e) Perforating arteries

89) Insulin increase absorbtion of glucose in?
a) Hepatocytes
b) Mucosa of small intestine
c) Renal tubular cells

90) Glucose is absorbed in renal tubules through:
a) Secondary Active transport
b) Facilitated diffusion
c) Diffusion


91) A pregnant women is on Phenytoin,wht is the least likely complication in fetus?
a) Bone deformity
b) Mental retardation
c) Cleft palate

92) Anticoagulants are indicated in all EXCEPT:
a) Thrombocytopenia
b) Cerebral infarction
c) Prolong bed rest
d) DVT

93) A mother brought her 8 yr old daughter in OPD with complain of vaginal bleeding,she was taller acc. To her age with well developed breasts.she was otherwise normal,wht is the cause:
a) Ovarian tumor
b) Hyperpituitrism
c) Hypothalamic disorder

94) Main function of Extracellular fluid is:
a) Transport electrolyte
b) Remove waste products
c) Provide nutrition

95) Which investigation is least likely done in a suspected case of DIC?
a) Clotting time
b) FDP
c) D-Dimer
d) Platelet count
e) PT

96) Which drug is not used in Dysmenorrhea?
a) Ibuprofen
b) Mefenamic acid
c) Aspirin
d) COX inhibitor


Questions
(Expect repetition of questions)

1)which hormone is resposible 4 storage of carbohydrate,fats,protein in body?
a)cortisol
b)growth hor
c)thyroxine

2)which hormone is responsible 4 storage of fat CHO n protein in body n called hormone of abundance?
a)costisol
b)throxine
c)grwoth hormone

3)which hormone is needed 4 brain development of fetous during intranatal life?
a)growth horm
b)thyroid
etc

4)tubo ovarian abcess in a woman using iucd is caued by?
a)actinomycosis
b)streptococi
etc

5)which component is low in cow milk as compare 2 mothr milk?
a)lactose
b)protien
c)calcium
d)fat

6)among following which is not antiseptic?
a)soap
b)salicylic acid
c)sodium hypochlorite
d)ethanol

7)4 tx of eclampsia which drug is used?
a)hydralazine
b)minoxidil
etc

8)which method is used 2 kill spore?
a)dry heat at 160c for 1 hr
b)dry heat at 160 c for 4 hr
c)moist heat at 160c for1hr

9)excess of aldosteron will cause?
a)hypokalemia
b)hyperkalemia
etc

10)structure of 2ndry villus unclude?
a)mesoderm cytotrophoblast syncytium
b)capillaries cytotrophoblast syncytium
etc

11)a 45 yrs old lady is post menopausal from last 9 months,which is most likely 2 show onset of menopause?
a)anovulatory cycle
b)atrophic uterus
etc

12)spermatogenesis is supported by?
a)FSH n testosteron
b)gonadotrophin
etc

13)germinal epithelium of ovary contain?
a)cuboidal epithelium
c)columnar
etc

14)columnar epithelium line which structure?
a)thymus
b)choroid plexus
c)uterine tubes

15)which method is most helpful in identifying chromosomal abnormalities?
a)karyotyping
b)u\s
etc

16)a normallooking female cam 4 primary amenorrhea on examination she was having short blind vagina wid normal vulva n scanty hair,her wt n ht for age were normal bt absence of uterus,she was othrwise a normal lookinggirl karyotype was xy
a)46xy
b)46xx
etc

17)DIC is related 2 all of the following except?
a)amniotic fluid embolism
b)cancer
c)gram-ve sepsis
d)retained dead fetus
e)etc(answer)

18)a women brought 2 hospital after having difficult delivery at home wid bleeding diathesis
wat is the cause
a)dic
etc etc

19)which of the following is leat likely 2 b effected by actinomycosis
a)bone
b)brain
c)oral cavity

19)which is not the manifestation of child born wid toxoplasma infection?
a)pneumonia
b)cataract
c)mental retardation
etc

20)a woman having morning stiffness of hand n pericardial effusion which lab finding woud help in dx?
a)anti nuclear DNA
b)strptolysin
c)rheumatic factor
etc

21)which structure pass thro inguinal canal from uterus 2 labium majus in female?
a)round ligament
b)broad ligament
etc

22)which ligament can b palpated on per rectal examination?
a)uterosacral
b)broad ligament
c)round ligament
etc

23)main support of uterus?
a)broad ligament
b)transverse cervical ligament
etc

24)an incision given supra pubically in a female for LSCS would damage which artery?
a)superior epigastric
b)inferior epigastric
etc

25)ovary is attached 2 back of braod ligament by?
a)mesovarium
etc etc

26)which sturcture does not form walls of inguinal canal?
a)lacunar
b)external oblique apenerosis
c)conjoint
etc

27)insulin help in absorption of glucose through?
a)musoca of small intestine
b)renal tubules
c)hepatocytes
etc

28)glucose is absorbed in renal tubules thro?
a)2ndry active transport
b)facilitated diffusion
etc

29)after lscs(c\section) a woman is bleeding intaperitonealy which artery can b damaged?
a)uterine
b)ovarian
c)internal iliac
etc

30)which organism require selective media 4 its diagnosis?
a)vibrio cholera
b)e coli
c)campylo bacter
etc

31)a pregnant woman on phenytoin, which is the least likely complication?
a)bone deformity
b)mental retardation
c)cleft palate

32)anticoagulant are contraindicated in pt wid?
a)thrombocytopenia
etc etc

33)a pt showing eosiniphilia,lymphoctytopenia polycythemia his blood levels show increase ACTH level which of following is the cause?
a)cushing syndrome
b)sudhing disease
c)addisons disease
etc

34)ca breast will effect which of the breast quadrant mostly?
a)upper n outer
b)medial
c)inner n medial
etc

35)a women having ca breast which group pf lypmh nodes is effected?
a)anterior axillay
etc

36)a mother brought her daughter wid vaginal bleeding she is 8 yrs old taller 4 her age wid well develop breast n odr wise normal which coud b the cause?
a)constitutional
b)hypothalamic
etc

37)a man wid full burns in icu develop bleeding diathesis which coud b the cause?
a)dic
b)infection
etc

38)a women brought 2 ER unconcious wid history of postpartum hemorrhage 10 yrs back which of the following is dx?
a)sheehan syndrome
b)low bp
etc

39) Main functions of extracellular fluid is?
a)transport electrolytes
b)removal of waste material
etc

40)therapeutic index show
a)drug efficacy
b)safety















More questions awaited, will be updated soon.
Related Posts Plugin for WordPress, Blogger...